CDM revision SEM 1 Flashcards

1
Q

What is the criteria for neutropenic sepsis and what. prophylaxis and treatment is given to a suspect or known case?

A

Neutropenic sepsis is a relatively common complication of cancer therapy, usually as a consequence of chemotherapy. It most commonly occurs 7-14 days after chemotherapy. It may be defined as a neutrophil count of < 0.5 * 109 in a patient who is having anticancer treatment and has one of the following:
a temperature higher than 38ºC or
other signs or symptoms consistent with clinically significant sepsis

Prophylaxis
if it is anticipated that patients are likely to have a neutrophil count of < 0.5 * 109 as a consequence of their treatment they should be offered a fluoroquinolone

Management
antibiotics must be started immediately, do not wait for the WBC
NICE recommends starting empirical antibiotic therapy with piperacillin with tazobactam (Tazocin) immediately
many units add vancomycin if the patient has central venous access but NICE do not support this approach
following this initial treatment patients are usually assessed by a specialist and risk-stratified to see if they may be able to have outpatient treatment
if patients are still febrile and unwell after 48 hours an alternative antibiotic such as meropenem is often prescribed +/- vancomycin
if patients are not responding after 4-6 days the Christie guidelines suggest ordering investigations for fungal infections (e.g. HRCT), rather than just starting therapy antifungal therapy blindly
there may be a role for G-CSF in selected patients

How well did you know this?
1
Not at all
2
3
4
5
Perfectly
2
Q

What are the risk factors for developing DVT?

A
General
increased risk with advancing age
obesity
family history of VTE
pregnancy (especially puerperium)
immobility
hospitalisation
anaesthesia
central venous catheter: femoral >> subclavian
Underlying conditions
malignancy
thrombophilia: e.g. Activated protein C resistance, protein C and S deficiency
heart failure
antiphospholipid syndrome
Behcet's
polycythaemia
nephrotic syndrome
sickle cell disease
paroxysmal nocturnal haemoglobinuria
hyperviscosity syndrome
homocystinuria

Medication
combined oral contraceptive pill: 3rd generation more than 2nd generation
hormone replacement therapy: the risk of VTE is higher in women taking oestrogen + progestogen preparations compared to those taking oestrogen-only preparations
raloxifene and tamoxifen
antipsychotics (especially olanzapine) have recently been shown to be a risk factor

How well did you know this?
1
Not at all
2
3
4
5
Perfectly
3
Q

What is the commonest non-Hodgkins lymphoma in the UK?

A

Diffuse large B cell lymphoma is by far the most common form of non-Hodgkin lymphoma (NHL) in the UK. It comprises around 40% of the total lymphoma cases. It is a high-grade tumour (i.e. aggressive) but typically responds better to treatment than some of the more indolent lymphomas.

How well did you know this?
1
Not at all
2
3
4
5
Perfectly
4
Q

How can Hodgkin’s and Non Hodgkin’s lymphoma be differentiated clinically?

A

While differentiating Hodgkin’s lymphoma from non-Hodgkin’s lymphoma is done by biopsy certain elements of the clinical presentation can help point towards one rather than the other.
Lymphadenopathy in Hodgkin’s lymphoma can experience alcohol-induced pain in the node
‘B’ symptoms typically occur earlier in Hodgkin’s lymphoma and later in non-Hodgkin’s lymphoma
Extra-nodal disease is much more common in non-Hodgkin’s lymphoma than in Hodgkin’s lymphoma

How well did you know this?
1
Not at all
2
3
4
5
Perfectly
5
Q

How can Lymphoma’s be Investigated and what is the characteristic biopsy finding in Burkitt’s lymphoma?

A

Excisional node biopsy is the diagnostic investigation of choice (certain subtypes will have a classical appearance on biopsy such as Burkitt’s lymphoma having a ‘starry sky’ appearance)
CT chest, abdomen and pelvis (to assess staging)
HIV test (often performed as this is a risk factor for non-Hodgkin’s lymphoma)
FBC and blood film (patient may have a normocytic anaemia and can help rule out other haematological malignancy such as leukaemia)
ESR (useful as a prognostic indicator)
LDH (a marker of cell turnover, useful as a prognostic indicator)
Other investigations can be ordered as the clinical picture indicates (LFT’s if liver metastasis suspected, PET CT or bone marrow biopsy to look for bone involvement, LP if neurological symptoms)

How well did you know this?
1
Not at all
2
3
4
5
Perfectly
6
Q

What are the laboratory findings in beta thalassemia?

A

The thalassaemias are a group of genetic disorders characterised by a reduced production rate of either alpha or beta chains. Beta-thalassaemia trait is an autosomal recessive condition characterised by a mild hypochromic, microcytic anaemia. It is usually asymptomatic

Features
mild hypochromic, microcytic anaemia - microcytosis is characteristically disproportionate to the anaemia
HbA2 raised (> 3.5%)
How well did you know this?
1
Not at all
2
3
4
5
Perfectly
7
Q

What are the types and causes of macrocytic anemia and what the features of megaloblastic anemia?

A

Macrocytic anaemia can be divided into causes associated with a megaloblastic bone marrow and those with a normoblastic bone marrow

Megaloblastic causes

vitamin B12 deficiency
folate deficiency

Normoblastic causes

alcohol
liver disease
hypothyroidism
pregnancy
reticulocytosis
myelodysplasia
drugs: cytotoxics

symptoms of anaemia, glossitis, a macrocytosis and hyper-segmented neutrophils on the blood film. This is typical of a megaloblastic anaemia such as B12 or folate deficiency anaemia. Another key feature of a megaloblastic anaemia is the presence of megaloblasts and giant metamyelocytes within the bone marrow. Following a full blood count, a blood film is the suggested second-line test for megaloblastic anaemia (as per BMJ best practice). It is also common practice to test for haematinics as this may reveal low B12 or folate levels and hence identify the cause of macrocytosis.

How well did you know this?
1
Not at all
2
3
4
5
Perfectly
8
Q

What are the causes of hyperkalemia and the ecg changes seen?

A

ECG changes seen in hyperkalaemia include tall-tented T waves, small P waves, widened QRS leading to a sinusoidal pattern and asystole

Causes of hyperkalaemia:
acute kidney injury
drugs*: potassium sparing diuretics, ACE inhibitors, angiotensin 2 receptor blockers, spironolactone, ciclosporin, heparin**
metabolic acidosis
Addison's disease
rhabdomyolysis
massive blood transfusion
How well did you know this?
1
Not at all
2
3
4
5
Perfectly
9
Q

What does raised ALP in the presence of normal LFT’s indicate?

A

Raised ALP in the presence of normal LFT’s should raise suspicion of malignancy. Particularly bone cancer/ metastases

How well did you know this?
1
Not at all
2
3
4
5
Perfectly
10
Q

How does hypophosphatemia present?how is it treated?

A

acute respiratory failure secondary to severe hypophosphataemia.

Hypophosphataemia is a common complication of insulin therapy in diabetic ketoacidosis (DKA) hence increasing the dosage of insulin is not an answer in this case. A rise in insulin causes phosphate to shift into the intracellular compartment, similar to the mechanism involved in hypophosphataemia as a result of refeeding syndrome or hyperglycaemic hyperosmolar non-ketotic coma (HONK).

Hypophosphataemia as a result of DKA treatment is usually transient and mild. Phosphate replacement therapy is rarely required unless it is severe and should be given as an infusion. Insulin therapy should never be stopped in a patient with DKA.

How well did you know this?
1
Not at all
2
3
4
5
Perfectly
11
Q

What are the causes and complications of hypophosphatemia?

A
Causes
alcohol excess
acute liver failure
diabetic ketoacidosis
refeeding syndrome
primary hyperparathyroidism
osteomalacia
Consequences
red blood cell haemolysis
white blood cell and platelet dysfunction
muscle weakness and rhabdomyolysis
central nervous system dysfunction
How well did you know this?
1
Not at all
2
3
4
5
Perfectly
12
Q

What are the causes of raised ALP?

A

Causes of raised alkaline phosphatase (ALP)
liver: cholestasis, hepatitis, fatty liver, neoplasia
Paget’s
osteomalacia
bone metastases
hyperparathyroidism
renal failure
physiological: pregnancy, growing children, healing fractures

The table below splits the causes according to the calcium level

Raised ALP and raised calcium	Raised ALP and low calcium
Bone metastases
Hyperparathyroidism
Osteomalacia
Renal failure
How well did you know this?
1
Not at all
2
3
4
5
Perfectly
13
Q

What are the causes of SIADH including drug causes?

A
Malignancy	
small cell lung cancer
also: pancreas, prostate
Neurological	
stroke
subarachnoid haemorrhage
subdural haemorrhage
meningitis/encephalitis/abscess
Infections	
tuberculosis
pneumonia
Drugs	
sulfonylureas*
SSRIs, tricyclics
carbamazepine
vincristine
cyclophosphamide
Other causes	
positive end-expiratory pressure (PEEP)
porphyrias
How well did you know this?
1
Not at all
2
3
4
5
Perfectly
14
Q

What are the causes of hypocalcemia and how can it be managed?

A

Causes
vitamin D deficiency (osteomalacia)
chronic kidney disease
hypoparathyroidism (e.g. post thyroid/parathyroid surgery)
pseudohypoparathyroidism (target cells insensitive to PTH)
rhabdomyolysis (initial stages)
magnesium deficiency (due to end organ PTH resistance)
massive blood transfusion
acute pancreatitis

Contamination of blood samples with EDTA may also give falsely low calcium levels.

Management
acute management of severe hypocalcaemia is with intravenous replacement. The preferred method is with intravenous calcium gluconate, 10ml of 10% solution over 10 minutes
intravenous calcium chloride is more likely to cause local irritation
ECG monitoring is recommended
further management depends on the underlying cause

How well did you know this?
1
Not at all
2
3
4
5
Perfectly
15
Q

How does Osmotic demyelination syndrome (central pontine myelinolysis) present?

A

can occur due to over-correction of severe hyponatremia
to avoid this, Na+ levels are only raised by 4 to 6 mmol/l in a 24-hour period
symptoms usually occur after 2 days and are usually irreversible: dysarthria, dysphagia, paraparesis or quadriparesis, seizures, confusion, and coma
patients are awake but are unable to move or verbally communicate, also called ‘Locked-in syndrome’

How well did you know this?
1
Not at all
2
3
4
5
Perfectly
16
Q

How is Hypercalcemia treated?

A

The initial management of hypercalcaemia is rehydration with normal saline, typically 3-4 litres/day. Following rehydration bisphosphonates may be used. They typically take 2-3 days to work with maximal effect being seen at 7 days

Other options include:
calcitonin - quicker effect than bisphosphonates
steroids in sarcoidosis

Loop diuretics such as furosemide are sometimes used in hypercalcaemia, particularly in patients who cannot tolerate aggressive fluid rehydration. However, they should be used with caution as they may worsen electrolyte derangement and volume depletion.

How well did you know this?
1
Not at all
2
3
4
5
Perfectly
17
Q

What can cause hypernatremia and how is it managed?

A
Causes of hypernatraemia
dehydration
osmotic diuresis e.g. hyperosmolar non-ketotic diabetic coma
diabetes insipidus
excess IV saline

Hypernatraemia should be corrected with great caution. Although brain tissue can lose sodium and potassium rapidly, lowering of other osmolytes (and importantly water) occurs at a slower rate, predisposing to cerebral oedema, resulting in seizures, coma and death1. Although there are no clinical guidelines by NICE or Royal College of Physicians at present, it is generally accepted that a rate of no greater than 0.5 mmol/hour correction is appropriate.

How well did you know this?
1
Not at all
2
3
4
5
Perfectly
18
Q

What new diabetic drug can potentially be used to treated renal impairment

A
  • In chronic kidney disease (CKD), dapagliflozin (Farxiga) reduced renal events and substantially improved overall survival, regardless of diabetes status, theDAPA-CKD trialshowed
How well did you know this?
1
Not at all
2
3
4
5
Perfectly
19
Q

How can CKD be classified?

A

Chronic kidney disease (CKD) can be defined by the presence of kidney damage or reduced kidney function for three or more months.
Chronic kidney disease (CKD) is classified using a combination of estimated glomerular filtration rate (eGFR) and urinaryalbumin:creatinine ratio(ACR).

  • Increased ACR is associated with increased risk of adverse outcomes.
  • Decreased GFR is associated with increased risk of adverse outcomes.
  • Increased ACR and decreased GFR in combination multiply the risk of adverse outcomes.
How well did you know this?
1
Not at all
2
3
4
5
Perfectly
20
Q

How can proteinuria be measured?

A

ACR>30

How well did you know this?
1
Not at all
2
3
4
5
Perfectly
21
Q

What is Alport’s syndrome?

A

Chronic kidney disease (CKD) is classified using a combination of estimated glomerular filtration rate (eGFR) and urinaryalbumin:creatinine ratio(ACR).

  • Increased ACR is associated with increased risk of adverse outcomes.
  • Decreased GFR is associated with increased risk of adverse outcomes.
  • Increased ACR and decreased GFR in combination multiply the risk of adverse outcomes.

COL4A5 is on thechromosome), so mutations in it can cause[X-linked])[Alport syndrome
COL4A3 and A4 are on[autosomes, meaning non-[sex chromosomes]and mutations in these cause either[autosomal recessive]

[Alport syndrome] which is also early onset, or[autosomal dominant][Alport syndrome](, which causes late onset disease.

• For diagnosis, Alport syndrome is typically suspected when there are clinical signs like gross hematuria or if there are vision or hearing problems, or microscopic hematuria with no apparent cause.

  • To confirm the diagnosis, a kidney or skin biopsy is often analyzed by immunohistochemistry, meaning a labelled antibody is applied to a biopsy sample on a slide.
  • The treatment for Alport syndrome usually focuses on the symptoms.
  • Proteinuria is treated with angiotensin converting enzyme inhibitors and/or angiotensin receptor blockers because there is evidence that this prevents progression to kidney failure.
  • Anterior lenticonus can be treated with a replacement lens, and Kidney failure necessitates dialysis or even kidney transplant.
How well did you know this?
1
Not at all
2
3
4
5
Perfectly
22
Q

What are the side effects of hemodialysis?

A
hypotension
arrhythmia
muscle cramps
nausea and vomiting 
headache
air embolism
seizure
disequilibrium syndrome
hemolysis
How well did you know this?
1
Not at all
2
3
4
5
Perfectly
23
Q

What are the complications for peritoneal dialysis

A
peritonitis
catheter non function
hyperlipidemia
risk of hernias
weight gain and hyperglycaemia
How well did you know this?
1
Not at all
2
3
4
5
Perfectly
24
Q
What are the 
Drug class 
Drug examples 
Mechanism of action 
SEs 
Contraindications 

of common oral hypoglycaemic agents?

A
Drug class 
Drug examples 
Mechanism of action 
SEs 
Contraindications 
Biguanide  
Metformin 

Other uses of metformin:
PCOS
NAFLD
Activates the AMP activated protein kinase (AMPK)
Increases insulin sensitivity
Decreases hepatic gluconeogenesis
GI side effects (can change to modified release if this is the case) - intolerable in 20% of people
Reduced vitamin b12 absorption
Lactic acidosis – this is rare. Occurs in severe liver disease/renal failure
No hypos, no weight gain
CKD – stop metofmrin if creatinine >150 or eGFR <30
Stop metformin if recent AKI, severe dehydration, recent MI (risk of lactic acidosis)
Sulfonylurea
Glomiperide, gliclazide
Increase pancreatic insulin secretion
Hypos
Increased appetite and weight gain

Rarer: 
SIADH 
Cholestatic liver dysfunction 
Peripheral neuropathy  
BM suppression 
Avoid in breast feeding and pregnancy 
Thiazolidinediones 
Pioglitazone 
Reduce peripheral insulin resistance 
Weight gain 
Liver impairment (LFTs need to be monitored) 
Fluid retention 
Bladder cancer 
Heart failure (due to fluid retention) 
DPP4-inhibitor 
Sitagliptin, alogliptin, vildagliptin - oral 
DPP4 inhibitors inhibit the breakdown of GLP1 (GLP1 = glucagon like peptide 1, it increases insulin secretin and inhibits glucagon secretion) 
Does not cause weight gain, does not cause hypos 

Pancreatitis

SGLT2- inhibitor
Empaglafiozine, dapagliflozin, canagliflozin
Inhibits sodium glucose co-transporter 2 (SGLT2) in the kidney – increases urinary glucose excretion
Risk of DKA – with normal/mildly elevated blood glucose—therefore need to warn patients of symptoms of DKA (e.g. Rapid weight loss, N, V, fast breathing, sleepiness, sweet breath)
Fouriner’s gangrene

If had previous DKA 
GLP 1 mimetic 
Exenatide – given Subcut 
Dulaglutide 
NICE guidance – used in obese patients only (BMI above 35) 

GLP1 is a hormone released by small intestine in response to glucose.
These drugs increase insulin secretion, inhibit glucagon secretion, slow gastric emptying
Weight loss (advantage)
Nausea, vomiting

How well did you know this?
1
Not at all
2
3
4
5
Perfectly
25
Q

How to separate pre-renal uraemia from acute tubular necrosis

A

Prerenal uraemia - kidneys hold on to sodium to preserve volume

How well did you know this?
1
Not at all
2
3
4
5
Perfectly
26
Q

What are the causes, signs and symptoms of hypokalaemia?

A

Symptoms include weakness, leg cramps, palpitations secondary to cardiac arrhythmias and ascending paralysis.

Causes can be secondary to:

1.) Increased potassium loss:
Drugs: thiazides, loop diuretics, laxatives, glucocorticoids, antibiotics
GI losses: diarrhoea, vomiting, ileostomy
Renal causes: dialysis
Endocrine disorders: hyperaldosteronism, Cushing’s syndrome

2.) Trans-cellular shift
Insulin/glucose therapy
Salbutamol
Theophylline
Metabolic alkalosis
  1. ) Decreased potassium intake
  2. ) Magnesium depletion (associated with increased potassium loss)

ECG changes seen in hypokalaemia include:
U waves
T wave flattening
ST segment changes

Treatment of hypokalaemia depends on severity. Any causative agents should be removed. Gradual replacement of potassium via the oral route is preferred if possible.

Mild to moderate hypokalaemia 2.5 - 3.4 mmol/l can be treated with oral potassium provided the patient is not symptomatic and there are no ECG changes.

Severe hypokalaemia (<2.5mmol/l) or symptomatic hypokalaemia should be managed with IV replacement. The patient should be managed in an area where cardiac monitoring can take place. If there are no contraindications to fluid therapy (e.g. volume overload, heart failure) potassium should be diluted to low concentrations as higher concentrations can be phlebitic. The infusion rate should not exceed 20mmol/hr. In this case, 3 bags of 0.9% Saline with 40mmol KCL is the correct answer.

How well did you know this?
1
Not at all
2
3
4
5
Perfectly
27
Q

What is Alpha 1 anti trypsin deficiency?How does it present and how is investigated and managed?

A

Genetics
located on chromosome 14
inherited in an autosomal recessive / co-dominant fashion*
alleles classified by their electrophoretic mobility - M for normal, S for slow, and Z for very slow
normal = PiMM
homozygous PiSS (50% normal A1AT levels)
homozygous PiZZ (10% normal A1AT levels)

Features
patients who manifest disease usually have PiZZ genotype
lungs: panacinar emphysema, most marked in lower lobes
liver: cirrhosis and hepatocellular carcinoma in adults, cholestasis in children

Investigations
A1AT concentrations
spirometry: obstructive picture

Management
no smoking
supportive: bronchodilators, physiotherapy
intravenous alpha1-antitrypsin protein concentrates
surgery: lung volume reduction surgery, lung transplantation

How well did you know this?
1
Not at all
2
3
4
5
Perfectly
28
Q

What are the causes of ascites

A

scites is the abnormal accumulation of fluid in the abdomen.

The causes of ascites can be grouped into those with a serum-ascites albumin gradient (SAAG) <11 g/L or a gradient >11g/L as per the table below:

SAAG > 11g/L
(indicates portal hypertension) SAAG <11g/L

Liver disorders are the most common cause
cirrhosis/alcoholic liver disease
acute liver failure
liver metastases

Cardiac
right heart failure
constrictive pericarditis

Other causes
Budd-Chiari syndrome
portal vein thrombosis
veno-occlusive disease
myxoedema

SAAG <11g/L

Hypoalbuminaemia
nephrotic syndrome
severe malnutrition (e.g. Kwashiorkor)

Malignancy
peritoneal carcinomatosis

Infections
tuberculous peritonitis

Other causes
pancreatitisis
bowel obstruction
biliary ascites
postoperative lymphatic leak
serositis in connective tissue diseases
How well did you know this?
1
Not at all
2
3
4
5
Perfectly
29
Q

What drugs are hepatotoxic?

A

Drug-induced liver disease is generally divided into hepatocellular, cholestatic or mixed. There is however considerable overlap, with some drugs causing a range of changes to the liver

The following drugs tend to cause a hepatocellular picture:
paracetamol
sodium valproate, phenytoin
MAOIs
halothane
anti-tuberculosis: isoniazid, rifampicin, pyrazinamide
statins
alcohol
amiodarone
methyldopa
nitrofurantoin

The following drugs tend to cause cholestasis (+/- hepatitis):
combined oral contraceptive pill
antibiotics: flucloxacillin, co-amoxiclav, erythromycin*
anabolic steroids, testosterones
phenothiazines: chlorpromazine, prochlorperazine
sulphonylureas
fibrates
rare reported causes: nifedipine

Liver cirrhosis
methotrexate
methyldopa
amiodarone

How well did you know this?
1
Not at all
2
3
4
5
Perfectly
30
Q

What is Budd chiari syndrome?

A

Budd-Chiari syndrome, or hepatic vein thrombosis, is usually seen in the context of underlying haematological disease or another procoagulant condition.

Causes
polycythaemia rubra vera
thrombophilia: activated protein C resistance, antithrombin III deficiency, protein C & S deficiencies
pregnancy
combined oral contraceptive pill: accounts for around 20% of cases

The features are classically a triad of:
abdominal pain: sudden onset, severe
ascites → abdominal distension
tender hepatomegaly

Investigations
ultrasound with Doppler flow studies is very sensitive and should be the initial radiological investigation

How well did you know this?
1
Not at all
2
3
4
5
Perfectly
31
Q

How can Ascites be managed

A

Management
reducing dietary sodium
fluid restriction is sometimes recommended if the sodium is < 125 mmol/L
aldosterone antagonists: e.g. spironolactone
loop diuretics are often added. Some authorities only add loop diuretics in patients who don’t respond to aldosterone agonists whereas other authorities suggest starting both types of diuretic on the first presentation of ascites
drainage if tense ascites (therapeutic abdominal paracentesis)-give albumin give prophylactic antibiotics(cipro)
large-volume paracentesis for the treatment of ascites requires albumin ‘cover’. Evidence suggests this reduces paracentesis-induced circulatory dysfunction and mortality

paracentesis induced circulatory dysfunction can occur due to large volume paracentesis (> 5 litres). It is associated with a high rate of ascites recurrence, development of hepatorenal syndrome, dilutional hyponatraemia, and high mortality rate
prophylactic antibiotics to reduce the risk of spontaneous bacterial peritonitis. NICE recommend: ‘Offer prophylactic oral ciprofloxacin or norfloxacin for people with cirrhosis and ascites with an ascitic protein of 15 g/litre or less, until the ascites has resolved’
a transjugular intrahepatic portosystemic shunt (TIPS) may be considered in some patients

How well did you know this?
1
Not at all
2
3
4
5
Perfectly
32
Q

What are the differentials for hepatic encephalopathy?

A

Patients may present similarly withacute alcoholic hepatitis

Other causes of confusion(which may be present in a patient with liver disease) and should always be considered when a patient with chronic liver disease presents confused

  • Delirium – which can be secondary to many causes but often infection
  • Head injury – subdural haematoma
  • Alcohol withdrawal
  • Drugs
How well did you know this?
1
Not at all
2
3
4
5
Perfectly
33
Q

What can cause an increase in conjugated and unconjugated bilirubin?

A

The commonest causes of raised unconjugated bilirubin in adults are:

  • Gilbert’s syndrome:
    • A genetic disorder where less of the enzyme that breaks down Bilirubin (UDP-glucuronyltransferase) is produced. It affects 5% of the population. There is an increase in the bilirubin often with fasting or concurrent illness.
    • Confirmation of just a predominant unconjugated hyperbilirubinaemia makes the diagnosis of Gilbert’s syndrome virtually certain. This does not require any treatment and the patient can be completely reassured.
    • Rarely does it cause a bilirubin above 68umol/L
  • Haemolysis:
    • The breakdown of red blood cells. There can by many causes for this and standard investigations for this would include: reticulocyte count, LDH, blood film, Haptoglobin, Direct coombs test
  • Drug related, for example, the antibiotic Rifampicin impairs the uptake of Bilirubin

In healthy adults conjugated bilirubin is virtually absent. Levels usually start to become increased when the liver has lost approximately ½ of its excretory capacity. It is therefore usually a sign of liver disease, which may be acute or chronic in nature.

Common causes for elevated conjugated bilirubin include:

  • Biliary obstruction at any level of the bile ducts
    • Often secondary to gallstones in the common bile duct, but also from malignancy – commonly Cholangiocarcinoma or Pancreatic cancer
  • Cholestatic drug reactions
    • Potentially any drug, but commonly antibiotics such as Nitrofuratoin and Penicillin
  • Autoimmune Cholestatic disease e.g. Primary Sclerosing Cholangitis (may occur at any level) and Primary Biliary Cirrhosis (intrahepatic ducts)
  • Hepatitis of any origin where there is significant impairment in liver function
  • Cirrhosis
How well did you know this?
1
Not at all
2
3
4
5
Perfectly
34
Q

What are triggers for chronic liver disease to become decompensated?

A

Causes of this acute:

The most likely cause of this presentation is that this man has decompensated liver disease with encephalopathy. Things which may may cause a patient to decompensate include

  • Upper GI bleed
  • Spontaneous bacterial peritonitis
  • Infection (any source)
  • Constipation
  • Drugs – prescription or recreational
  • Dehydration
  • Alcohol – either a binge or withdrawal
  • Portal vein thrombosis
How well did you know this?
1
Not at all
2
3
4
5
Perfectly
35
Q

What are the features of PBC and how is it treated?

A

common in female-damage to intrahepatic ducts
Presentation-no symptoms,itch,Sjorens,jaundice ,fatigue,poor memory

Labs-raised ALP ,raised IgM,AMA,biopsy-granulomatous lymphatic cholangitis(not needed-florid ducts, duct loss)
Exclude other causes

treatment-UDCA
Itch-Cholestyramine, rifampicin, gabapentin

Treat symptoms-fatigue

Monitering-itch,osteoporosis,cirrhosis

liver transplant

How well did you know this?
1
Not at all
2
3
4
5
Perfectly
36
Q

How does autoimmune hepatitis present and how is it diagnosed?

A

Any age
Can be precipitated by drugs such as statins, statins,nitrofurantoin

types

type 1-80%(ANA/ASMA)
type 2-<10%(liver kidney microsomal LKM,liver cystosol antibody-high risk outcome

presentation-fatigue,aneroxia,nausea,joint pain, acute hepatitis jaundice

labs
raised ALT
ANA/ASMA
RAISED IgG
usually need biopsy confirm fibrosis

Treatment-aim for normal ALT and IgG
lifelong immunosupression
-predinsolone,Azathioprine,MMF,tacrolimus,liver transplant

How well did you know this?
1
Not at all
2
3
4
5
Perfectly
37
Q

How does PBC present and how is it diagnosed and managed?

A

Inflammation and fibrosis of both intra and extra hepatic bile ducts

70-80% have IBD(UC)

symptoms-asymptomatic,fatique,itch,RUQ pain,cholangitis,jaundice and complication

labs-raised ALP,MRCP shows beads on string appearance
CT not helpful US normal

biopsy-onion skin

treatment

IBD-colonoscopy
Itch-same as PBC
manage cholangitis if causing symptoms

Complications
bone disease
cholangiocarcinoma (in 10%)
increased risk of colorectal cancer

liver transplantation

How well did you know this?
1
Not at all
2
3
4
5
Perfectly
38
Q

What causes Wilson’s disease how does it present and how is it managed?

A

Wilson’s disease is a rare progressive genetic disorder, which results in accumulation of copper in the body’s tissues. In particular it affects the brain, liver and cornea of the eyes. Untreated it may lead to liver fibrosis and cirrhosis, along with central nervous system dysfunction.

There are 500 gene mutations known about. These result in dysfunction in Wilson’s ATPase(ATP7B-helps Cu bind to ceruloplasmin and form vesicles to be excreted in the bile), which usually moves copper from along intracellular membranes in the liver. The defective ATPase results in accumulation of copper in the liver, resulting in damage. Over time as the liver becomes damaged the copper is released into the blood and causes other end organ damage.

Patients may present with:

Liver dysfunction (most common presentation):

·Decompensated liver cirrhosis

·Acute liver failure (often associated with renal failure and haemolytic anaemia)

Central nervous system (usually have established liver disease):

·Asymmetrical tremor in ½ patients with CNS dysfunction

·Poor co-ordination and clumsiness

·Speech and language problems

·Neuropsychiatric illness – commonly severe depression or neuroticbehaviours

Enlarged liver and spleen

Renal disease due to damage to distal renal tubules

hemolytic anemia

Ophthalmological

·Kayser-Fleischer ring present in 95% of those with neurological disease (seen on slit-lamp), 50% of those without. May occur in other diseases.

·Sun-flower cataracts – seen on slit lamp. Do not impair vision

Diagnosis.

Consider in any patient of any age with unusual liver or neurological abnormalities. There is no one best test for diagnosis.

Presence of Kayser-Felischer ring and low serum caeruloplasmin (<0.1g/L) is enough to establish diagnosis.

Caeruloplasmin is a protein made in the liver that stores and carries copper around the blood. This may be low in Wilson’s as copper is not able to bind to the protein causing instability. Serum Caeruloplasmin is can be affected by many factors and is not diagnostic on it’s own.

Other markers of disease include 24hour urinary copper, serum free copper and hepatic copper (liver biopsy).

A scoring system also exists to help with diagnosis.

Genetic screening is complex, many patients are compound heterozygotes (carry two different defective genes) and it can take many months to complete.
Treatment

Treatment is life-long. It involves using medication to either promote urinary excretion or to decrease intestinal absorption. The commonest medication used is D-Penicillamine.

Zinc salts,trientine,tetrathiomolybdate

Genetics are complex due to the number of gene variations, but essentially siblings (25% risk) and any offspring (0.5% risk) should be offered genetic screening.

How well did you know this?
1
Not at all
2
3
4
5
Perfectly
39
Q

What are grades of hepatic encephalopathy and how is it treated?

A

I-irritabilty
ii-confusion
iii-incoherent inappropriate behaviour, restless
iv-coma

Precipitating factors
infection e.g. spontaneous bacterial peritonitis
GI bleed
post transjugular intrahepatic portosystemic shunt
constipation
drugs: sedatives, diuretics
hypokalaemia
renal failure
increased dietary protein (uncommon)

Management
treat any underlying precipitating cause
NICE recommend lactulose first-line, with the addition of rifaximin for the secondary prophylaxis of hepatic encephalopathy
lactulose is thought to work by promoting the excretion of ammonia and increasing the metabolism of ammonia by gut bacteria
antibiotics such as rifaximin are thought to modulate the gut flora resulting in decreased ammonia production
other options include embolisation of portosystemic shunts and liver transplantation in selected patients

How well did you know this?
1
Not at all
2
3
4
5
Perfectly
40
Q

What are the causes of Acute liver failure?

A

ALF-HE defining feature

– drugs (inc paracetamol, abx and anti-epileptics), viruses (typically hepatitis A, B and E), toxins (classically mushroom poisoning, many herbal remedies are implicated), vascular causes (e.g. budd chiari or ischaemic/hrpoxic hepatitis), pregnancy-related and some miscellaneous causes e.g. acute and fulminant presentation of Wilson disease/AIH) and malignant infiltration of the liver, typically by breast cancer or lymphoma

How well did you know this?
1
Not at all
2
3
4
5
Perfectly
41
Q

What is the King college criteria for Emergency liver transplant?

A

ALF due to paracetamol

Arterial pH <7.3 after resuscitation and>24 hours since ingestion
Lactate >3 mmol/L or
The 3 following criteria:
HE >Grade 3
Serum creatinine >300 µmol/L
INR >6.5

ALF not due to paracetamol

INR >6.5 or
3 out of 5 following criteria:
Aetiology: indeterminate aetiology, hepatitis, drug-induced hepatitis
Age <10 years or >40 years
Interval jaundice encephalopathy >7 days
Bilirubin >300 µmol/L
INR >3.5
How well did you know this?
1
Not at all
2
3
4
5
Perfectly
42
Q

What are the contraindications for liver transplant?

A

Medical

Untreated or progressive infection

Clinically apparent extrahepatic or metastatic malignancy

Progressive hypotension, resistant to vasopressor support

Clinically significant ARDS, FiO2 > 0.8

Fixed dilated pupils > 1 hour in the absence of thiopentone

Severe coexistent cardiopulmonary disease

HIV?

psychiatric

Multiple episodes of self harm (>5) within an established pattern of behaviour (esp. if non-drug methods used)

Consistently stated wish to die, in the absence of established mental illness

Chronic refractory schizophrenia or other mental illness, resistant to therapy

Incapacitating dementia or mental retardation

Active intravenous drug abuse or oral polydrug use

Alcohol dependence or abuse

How well did you know this?
1
Not at all
2
3
4
5
Perfectly
43
Q

What is the pathophysiology of Alcoholic liver disease?

A

Ethanol is metabolized in the liver by two pathways, resulting in an increase in the NADH/NAD ratio. The altered redox potential causes increased hepatic fatty acid synthesis with decreased fatty acid oxidation; both events lead to hepatic accumulation of fatty acid,which is then esterified to glycerides.
The changes in oxidation–reduction also impair carbohydrate and protein metabolism and are the cause of the centrilobular necrosis of the hepatic acinus that is typical of alcohol damage. TNFα release from Kupffer cells causes the release of reactive oxygen species,leading, in turn, to tissue injury and fibrosis.Acetaldehyde is formed by the oxidation of ethanol, and its effect on hepatic proteins may
well be a factor in producing liver cell damage.

n addition to fatty change, there is infiltration by polymorphonuclear leucocytes and
hepatocyte necrosis, mainly in zone 3. Dense cytoplasmic inclusions called Mallory bodies are
sometimes seen in hepatocytes and giant mitochondria are also a feature. Mallory bodies are
suggestive of, but not specific for, alcoholic damage, as they can be found in other liver
disease, such as Wilson’s disease and primary biliary cholangitis. If alcohol consumption
continues, alcoholic hepatitis may progress to cirrhosis.

How well did you know this?
1
Not at all
2
3
4
5
Perfectly
44
Q

How does Alcoholic liver disease present?

A

Alcoholic liver disease covers a spectrum of conditions:
alcoholic fatty liver disease
alcoholic hepatitis
cirrhosis

tender hepatomegaly
Selected investigation findings:
gamma-GT is characteristically elevated
the ratio of AST:ALT is normally > 2, a ratio of > 3 is strongly suggestive of acute alcoholic hepatitis
Hypoglycemias,Increased MCV,thrombocytopenia

Selected management notes for alcoholic hepatitis:
glucocorticoids (e.g. prednisolone) are often used during acute episodes of alcoholic hepatitis
Maddrey’s discriminant function (DF) is often used during acute episodes to determine who would benefit from glucocorticoid therapy
it is calculated by a formula using prothrombin time and bilirubin concentration
pentoxyphylline is also sometimes used
the STOPAH study (see reference) compared the two common treatments for alcoholic hepatitis, pentoxyphylline and prednisolone. It showed that prednisolone improved survival at 28 days and that pentoxyphylline did not improve outcomes

How well did you know this?
1
Not at all
2
3
4
5
Perfectly
45
Q

How is Haemochromatosis inherited and what are the genes involved?

A

The two known mutations of HFE are C282Y and H63D. Testing for these is simple and cheap. But there are other rare genes that may cause disease

Whilst this is an autosomal recessive condition, clinically things can be a bit more complex – this is because gene expression (penetrance) varies. E.g. just because you have two copies of the defective gene does not mean you will definitely develop the clinical condition.

  • C282Y/C282Y Homozygous gives a 95% risk of iron overload
  • C282Y/H63D compound heterozygotes gives around 4% risk (increased risk with increased alcohol intake, viral hepatitis)
  • H63D/H63D Homozygous is least likely to cause iron overload

All these genes lead to increased intestinal absorption of iron.

How well did you know this?
1
Not at all
2
3
4
5
Perfectly
46
Q

What are the features of haemochromatosis?

A

resenting features
early symptoms include fatigue, erectile dysfunction and arthralgia (often of the hands)
‘bronze’ skin pigmentation
diabetes mellitus
liver: stigmata of chronic liver disease, hepatomegaly, cirrhosis, hepatocellular deposition)
cardiac failure (2nd to dilated cardiomyopathy)
hypogonadism (2nd to cirrhosis and pituitary dysfunction - hypogonadotrophic hypogonadism)
arthritis (especially of the hands)

Increase risk if pseudo put

Questions have previously been asked regarding which features are reversible with treatment:

Reversible complications	Irreversible complications
Cardiomyopathy
Skin pigmentation
Liver cirrhosis**
Diabetes mellitus
Hypogonadotrophic hypogonadism
Arthropathy

Typical iron study profile in patient with haemochromatosis
transferrin saturation > 55% in men or > 50% in women
raised ferritin (e.g. > 500 ug/l) and iron
low TIBC

How well did you know this?
1
Not at all
2
3
4
5
Perfectly
47
Q

How can NAFLD be identified and how id the fibrosis staged?

A

Who is an risk?
All overweight/obese individuals (50% of population)
Older sedentary individuals
Type 2 diabetics/metabolic syndrome
Often goes undiagnosed as is it frequently asymptomatic and raised liver enzymes frequently ignored
How are patients with NAFLD typically identified?

Raised liver enzymes
Imaging evidence of steatosis
Present with imaging evidence of cirrhosis/liver cancer (too late!)

dIAGNOSIS

Raised ALT and/or GGT and evidence of steatosis on imaging

Imaging evidence of steatosis

Raised ALT and/or GGT and evidence of insulin resistance / central obesity / metabolic risk factors

No history of excess alcohol (<14/21 units/week)
No known pre-existing liver disease
No hepatotoxic drugs
Negative blood screen for other liver diseases (viral, autoimmune etc)

Liver biopsy may be required if there is diagnostic uncertainty

xRaised IgA in 46%
Raised ferritin with normal transferrin saturation in 33%

Staging for fibrosis
FIB-4 score
NAFLD fibrosis score
ELF test
Fibrotest
ProC3

Transient elastography
Acoustic force radiation imaging (ARFI)
MR elastography

Generally tests are reasonably good at differentiating patients in to no/mild fibrosis
or advanced fibrosis/cirrhosis, but inaccurate in providing an exact fibrosis stage

How well did you know this?
1
Not at all
2
3
4
5
Perfectly
48
Q

How can NAFLD be managed?

A
General
Lifestyle intervention (aim to lose >10%  bodyweight)
Specialist diet/exercise advice
Weight loss adjuncts (orlistat)
Modify cardiovascular risk
Bariatric surgery (if meet criteria)

F0-F1

As above
Fibrosis reassessment every 3 years

F2-F3
Non-diabetic 
   Vitamin E
   Clinical trial
Diabetic 
  Regimen including GLP-1 or Pioglitazone or empagliflozin
  Clinical trial
Cirrhosis
Surveillance for HCC
Variceal screening
Bone density assessment
Clinical trial
How well did you know this?
1
Not at all
2
3
4
5
Perfectly
49
Q

What follow up is usually given to a patient with Cirrhosis and decompensation?

A

Suggested medical follow up:

  • Hepatocellular carcinoma: screening ultrasound and AFP every 6 months
  • Variceal Haemorrhage: OGD at diagnosis of cirrhosis and every 2 years
  • Beta-blocker (non-cardio-selective; carvedilol or propranol) as primary prevention of bleedingrebleed
  • Viral Superinfection: Immunise for HAV and HBV (although this patient has already been exposed to HBV therefore would not require additional vaccination)
  • Osteoporosis: Screen & treat
  • Ascites & SBP: Monitor regularly for evidence of ascites & treat
  • Vitamin deficiencies: Treat (Vit B Co-strong & Thiamine)
  • Ongoing review with addiction services to help maintain abstinence from alcohol and illicit drugs.
How well did you know this?
1
Not at all
2
3
4
5
Perfectly
50
Q

What Scoring systems can be used to determine the severity of liver Cirrhosis?

A

For many years the Child-Pugh classification was used to classify the severity of liver cirrhosis. However, in recent years the Model for End-Stage Liver Disease (MELD) has been increasingly used, particularly patient’s who are on a liver transplant waiting list

Child-Pugh classification

Score	1	2	3
Bilirubin (µmol/l)	<34	34-50	>50
Albumin (g/l)	>35	28-35	<28
Prothrombin time,
prolonged by (s)	<4	4-6	>6
Encephalopathy	none	mild	marked
Ascites	none	mild	marked

Summation of the scores allows the severity to be graded either A, B or C:
< 7 = A
7-9 = B
> 9 = C

MELD

Uses a combination of a patient’s bilirubin, creatinine, and the international normalized ratio (INR) to predict survival. A formula is used to calculate the score:

MELD = 3.78×ln[serum bilirubin (mg/dL)] + 11.2×ln[INR] + 9.57×ln[serum creatinine (mg/dL)] + 6.43

The 3-month mortality based on MELD scores:
40 or more: 71.3% mortality
30 - 39: 52.6% mortality
20 - 29: 19.6% mortality
10 - 19: 6.0% mortality
< 9: 1.9% mortality
How well did you know this?
1
Not at all
2
3
4
5
Perfectly
51
Q

What are the indications for liver transplant in the UK?

A
Chronic liver failure
UKELD ≥ 49
(Specific variant syndromes)
Hepatocellular carcinoma
multiple: up to 5 tumours ≤ 3cm
single: up to 5cm (or 7cm if stable)
(Acute liver failure)
How well did you know this?
1
Not at all
2
3
4
5
Perfectly
52
Q

What scoring systems are used in Alcoholic hepatitis?

A

Several models are available to determine the severity of alcoholic hepatitis.
The Maddrey discriminant function (DF), Model for End-stage Liver Disease (MELD) and Glasgow alcoholic hepatitis score (GAH) can all be used to assess the severity of alcoholic hepatitis. These scores are predominantly based on laboratory parameters.

Determining severity of alcoholic hepatitis is important to highlight patients with poor short-term survival and those who would benefit from pharmacological intervention.

Maddrey discriminant function

The DF score has been traditionally use to assess the severity of alcoholic hepatitis. It is based on serum bilirubin and prothrombin time.

DF = (4.6 x [prothrombin time (sec) - control prothrombin time (sec)]) + (serum bilirubin/17.1)

Serum bilirubin (umol/L)

Severe alcoholic hepatitis is defined as a DF score ≥ 32. The 28 day (one month) mortality among patients with a DF ≥32 ranges from 25-45%. Patients with a score < 32 have mild-to-moderate alcoholic hepatitis, which has a <10% mortality at 1-3 months.

Glasgow alcoholic hepatitis score

The GAH is a newer scoring system, which also predicts mortality among patients with alcoholic hepatitis. It is a slightly more complex score based on age, white blood cell count, urea, bilirubin and prothrombin time.

A score ≥ 9 is consistent with severe alcoholic hepatitis and associated with a poor 28-day and 84-day survival (46% and 40%, respectively).

How well did you know this?
1
Not at all
2
3
4
5
Perfectly
53
Q

What are the causes of an Upper Gi bleed?

A

Oesophagus

Oesophagitis
Varices
Malignancy
Gastro-oesophageal reflux disease (GORD)
Mallory-Weiss tear
Stomach
Peptic ulcer disease
Mallory-Weiss tear
Gastric varices
Gastritis
Malignancy
Duodenum
Peptic ulcer disease
Diverticulum
Aortoduodenal fistula
Duodenitis
Other

There are many other causes of upper GI bleeding though they are relatively less common.

Swallowed blood
Bleeding disorders
Dieulafoy’s lesion
Aortoenteric fistula
Hereditary haemorrhagic telangiectasia (Osler-Weber-Rendu syndrome)
Gastric antral vascular ectasia (GAVE, watermelon stomach)

How well did you know this?
1
Not at all
2
3
4
5
Perfectly
54
Q

What are the risk factors and features of an upper Gi bleed?

A
Risk factors
Several risk factors increase the likelihood of a patient developing an UGIB.
NSAIDs
Anticoagulants
Alcohol abuse
Chronic liver disease
Chronic kidney disease
Advancing age
Previous PUD or H. pylori infection
NSAID-use

NSAIDs inhibit the synthesis of prostaglandins, which are gastroprotective.

Prostaglandins work by inhibiting enterochromaffin-like cells, which are involved in the secretion of histamine. Histamine stimulates parietal cells to secrete hydrochloric acid. Therefore, inhibition of prostaglandins leads to excessive HCl secretion and damage to the underlying mucosa.

Clinical features
The two characteristic clinical findings of UGIB are haematemesis and melaena.
Haematemesis refers to vomiting blood. Haematemesis may present with bright red bleeding or as ‘coffee-ground’ vomitus.

Melaena is the passage of ‘black, tarry stool’, which has an offensive odour. The colour is due to partly digested blood.

Symptoms

Haematemesis
Dizziness
Syncope
Weakness
Abdominal pain
Dyspepsia
Heartburn
Melaena
Haematochezia (passage of fresh blood per rectum)
Weight loss
Signs
Dehydration
Pallor
Confusion
Tachycardia and hypotension
Abdominal tenderness
Melaena
Haematochezia (10-15% of patients with acute, severe haematochezia have an UGIB)
Stigmata of liver disease (e.g. spider naevi, ascites, hepatomegaly)
Telangiectasia
How well did you know this?
1
Not at all
2
3
4
5
Perfectly
55
Q

What is the main investigation done in upper gi

bleed and what scoring system is used?

A

Upper GI endoscopy is the main diagnostic test. It should be completed immediately in any unstable patient following initial resuscitation, or within 24 hours in all other patients.
arly risk stratification helps identify high-risk patients & need for prompt intervention. Two scoring systems are used in patients presenting with UGIB.
UGIB scoring systems

Blatchford score

The Blatchford score takes into account a number of different clinical findings, biochemical parameters and past medical history. NICE recommends it is used during the primary assessment, followed by the Rockall score post-endoscopy.

Similar to the Rockall score, a score of 0 on the Blatchford score is associated with a low risk of mortality and patients can be considered for early discharge and non-admission.

Rockall scoring

The Rockall score is comprised of both a pre- and post-endoscopy score that can be added together to give an overall value.

The pre-endoscopy score is composed of three parts:

Age (0-2)
Shock (0-2)
Co-morbidity (0-3)
Patients with a score of 0 are at low risk of re-bleeding and death. This group of patients (approx. 15%) may be discharged early or not admitted.

The post-endoscopy score is composed of two sections:

Diagnosis (0-2)
Bleeding (0, 2)
This score can be remembered using the mnemonic ABCDE:

Age
Blood pressure (and heart rate)
Comorbidity
Diagnosis
Endoscopic findings
How well did you know this?
1
Not at all
2
3
4
5
Perfectly
56
Q

How can an Upper Gi bleed be managed?

A

All patients presenting with an UGIB should initially be resuscitated with respects to airway (A), breathing (B) and circulation (C).
Resuscitation

Airway
Patent and protected
Breathing
Saturations
Respiratory rate
Breath sounds
Circulation
Blood pressure & heart rate
ECG
Establish IV access (two wide-bore cannula)
Take blood (e.g. FBC, U&Es, clotting, LFTs, cross match)
IV fluid if appropriate (0.9% normal saline)
Consider blood products
Endoscopy

Following resuscitation, unstable patients should be transferred for an immediate endoscopy. It is recommended that all other patients have endoscopy within 24 hours of admission.

Management can then be divided according to ‘non-variceal’ or ‘variceal’ bleeding.

Non-variceal bleeding

The most common cause of non-variceal upper GI bleeding is peptic ulcer disease. The need for intervention depends on the characterisation of the ulcer, which can be classified using the Forrest classification (beyond the scope of these notes). Management below focuses mainly on that of peptic ulcer disease. Alternative pathologies may be treated slightly differently (e.g. argon photocoagulation for angiodysplasia).

A number of techniques can be employed at the time of endoscopy to treat non-variceal causes of UGIB. In general, dual therapy should be given (i.e. adrenaline plus another modality)

Mechanical (e.g. clips) with adrenaline
Thermal coagulation with adrenaline
Proton pump inhibitor therapy should be reserved for patients with a non-variceal UGIB with evidence of recent haemorrhage during endoscopy.

A repeat endoscopy should be completed in patients who re-bleed, or are suspected to be high-risk of re-bleeding. Unstable patients who re-bleed post-endoscopy should be offered radiological (e.g. embolisation) or surgical intervention.

Variceal bleeding

Pharmacological intervention

Terlipressin (IV injection)
Analogue of vasopressin (ADH)
Causes splanchnic vasoconstriction
This reduces portal pressures
Prophylactic antibiotic therapy 
Reduces the risk of spontaneous bacterial peritonitis
Endoscopic intervention

Variceal band ligation (VBL)
Completed acutely. Patients then need to undergo variceal banding programme every 2-4 weeks until varices have gone.
Endoscopic sclerotherapy
Alternative option to VBL that involves injection of a sclerosing agent.
Failed intervention

Patients may re-bleed despite endoscopic therapy. An initial re-attempt of variceal band ligation may be appropriate. If these attempts fail, further options include:

Sengstaken-blakemore tube:
Bridging therapy, at risk of oesophageal necrosis if left > 24 hours.
Oesophageal stent:
Alterantive to Sengstaken-blakemore tube.
Transjugular intrahepatic portosystemic shunt (TIPS) procedure:
Interventional radiological procedure to create a shunt between portal and systemic venous circulation to reduce portal pressure.
A definitive treatment in appropriately selected patients.

How well did you know this?
1
Not at all
2
3
4
5
Perfectly
57
Q

What is TIPPS and what is a complication?

A

HE

How well did you know this?
1
Not at all
2
3
4
5
Perfectly
58
Q

What are the causes of Hypertension?

A
  • Renal disorders are the most common cause of secondary hypertension. They include:
    • Chronic pyelonephritis
    • Diabetic nephropathy
    • Glomerulonephritis
    • Polycystic kidney disease
    • Obstructive uropathy
    • Renal cell carcinoma
  • Other causes ofsecondary hypertension are:
    • Vascular disorders, including:
      • Coarctation of the aorta— usually results in upper-limb hypertension. There can be a significant difference in blood pressure between the left and right arms. Other signs include absent or weak femoral pulses, radio-femoral delay, palpable collateral blood vessels in the back muscles, and a suprasternal murmur radiating through to the back.
      • Renal artery stenosis— suspect this if the person has peripheral vascular disease and an abdominal bruit, or if blood pressure is resistant to treatment.
    • Endocrine disorders, including:
      • Primary hyperaldosteronism— probably the most common curable cause of hypertension. People usually present with hypokalaemia, alkalosis (elevated bicarbonate level), and plasma sodium level greater than 140mmol/L, or a larger than expected decrease in serum potassium when using a low-dose thiazide-type diuretic. The symptoms may be non-specific, but rarely it may present with tetany, muscle weakness, nocturia, or polyuria. Treatment with a calcium-channel blocker can mask the features of primary hyperaldosteronism. After identification of a possible adrenal adenoma on CT scan or MRI (magnetic resonance imaging), tertiary referral is required for confirmation of unilateral aldosterone excess and possible laparoscopic adrenalectomy.
      • Phaeochromocytoma— people can present with intermittently high or labile blood pressure, or postural hypotension, headaches, sweating attacks, palpitations, or unexplained fever and abdominal pains. Alternatively, it can be asymptomatic. It is the rarest but most important cause of hypertension to diagnosebecause malignant transformation or catastrophic haemorrhage from the tumour can be fatal.
      • Cushing’s syndrome— suspect this when clinical features are present (for exampletruncal obesity and striae). It rarely presents as hypertension alone.
      • Acromegaly— suspect this if clinical features are present (for exampleenlargement of hands and feet, facial changes, sweating).
      • Hypothyroidism — hypertension may result from altered levels of renin, angiotensin, and aldosterone, and is associated with an increased diastolic blood pressure. Clinical features may include fatigue, weight gain, dry skin and hair loss, constipation, and muscle weakness. See the CKS topic onHypothyroidismfor more information.
      • Hyperthyroidism — increased systolic blood pressure may result. Clinical features may include tremor, anxiety, sweating, weight loss, diarrhoea, and heat intolerance. See the CKS topic onHyperthyroidismfor more information.
    • **
      • Connective tissue disorders (scleroderma, systemic lupus erythematosus, polyarteritis nodosa).
      • Retroperitoneal fibrosis.
      • Obstructive sleep apnoea. See the CKS topic onObstructive sleep apnoea syndromefor more information.
How well did you know this?
1
Not at all
2
3
4
5
Perfectly
59
Q

What medication can causes hypertension?

A

Drugs and other substances, including:**

    - Alcohol— misuse of alcohol may be the most common individual secondary cause of hypertension. Features include variable hypertension that is resistant to commonly used drugs and that disappears within a week or two of complete abstinence.
    - Ciclosporin.
    - Cocaine and other substances of abuse.
    - Combined oral contraceptive.
    - Corticosteroids.
    - Erythropoietin.
    - Leflunomide.
    - Liquorice— present in some herbal medicines.
    - Nonsteroidal anti-inflammatory drugs.
    - Sympathomimetics— may be found in over-the-counter cough and cold remedies (for example ephedrine andphenylpropanolamine).
    - Venlafaxine.
- **Other conditions, including:**
How well did you know this?
1
Not at all
2
3
4
5
Perfectly
60
Q

How does Hypertensive retinopathy present?

A

Hypertension may cause progressive retinal microvascular changes.
These changes have been classified by the Keith-Wagener Barker (KWB) grades:

Grade 1: Generalised arteriolar narrowing (silver wiring).
Grade 2: Focal narrowing and arteriovenous nipping.
Grade 3: Retinal haemorrhages, cotton wool spots (retinal nerve fibre layer micro-infarcts leading to exudation of axoplasmic materials).
Grade 4: Papilloedema
Grade may indicate malignant hypertension requiring admission and immediate management. Recently there has been a move away from the KWB grades with a new three stage system proposed.

How well did you know this?
1
Not at all
2
3
4
5
Perfectly
61
Q

How can Hypertensive emergencies be managed?

A

Hypertensive emergencies occur when high BP results in acute end-organ damage.
The term malignant (or accelerated) hypertension is typically reserved for when papilloedema is present and is defined by NICE as:

A BP >180/120 with signs of papilloedema and/or retinal haemorrhage.

It is a severe condition resulting in neurological, renal and cardiac damage, requiring admission and immediate management.

Treatment attempts to reduce BP over 24-48hrs. This is to prevent hypoperfusion. Changes may have occurred to autoregulatory mechanisms of blood pressure control. Therefore, a rapid reduction in blood pressure, even to normal levels, may result in profound organ hypoperfusion.

Therapies include:

IV Nitroprusside (a nitric oxide releasing drug), labetalol and glyceryl trinitrate infusions are options.
Phentolamine (alpha-adrenergic antagonist) also used in phaeochromocytoma crisis.
How well did you know this?
1
Not at all
2
3
4
5
Perfectly
62
Q

What are the blood pressure targets for different populations?

A

Patients < 80 years: clinic BP < 140/90 mmHg / ABPM < 135/85 mmHg
Patients ≥ 80 years: clinic BP < 150/90 mmHg / ABPM < 145/85 mmHg
Those with renal disease and proteinuria or diabetes should target a BP of < 130/80 mmHg.

How well did you know this?
1
Not at all
2
3
4
5
Perfectly
63
Q

How can hypertension complications be managed?Who is treated?

A

​Management of hypertension is based upon NICE guidelines.
Modifiable risk factors

Lifestyle modification & patient education are important in treating hypertension.

Offer advice that targets the patient’s modifiable risk factors.
Discourage excessive caffeine and alcohol, if appropriate offer smoking cessation advice.
Consider the need for anti-platelets or a statin.
Whom to treat

If clinic BP < 140/90 mmHg or ABPM < 135/85 mmHg, check BP at least every 5 years or more often if clinic BP close to 140/90 mmHg. If evidence of end-organ damage, consider other causes.

Antihypertensive drug therapy is initiated in patients:

Aged < 80 years with stage 1 hypertension and with one of the following; end organ damage, cardiovascular disease, renal disease, diabetes or 10-year cardiovascular risk ≥10%.
of any age with stage 2 hypertension
of any age with stage 3 hypertension (consider immediate treatment)
Consider treatment in patients > 80 years old with stage 1 hypertension if clinic BP is > 150/90 mmHg. However, take into account frailty and co-morbidities. Patients < 60 years with stage 1 hypertension can be considered for antihypertensive therapy even if the 10-year cardiovascular risk < 10%.

Specialist assessment

Patients with stage 3 hypertension (≥ 180/120 mmHg) should be referred for same-day specialist assessment if any features of accelerated hypertension are identified:

New onset confusion
Chest pain
Signs of heart failure (e.g. shortness of breath, fluid overload)
Acute kidney injury
Papilloedema
Retinal haemorrhage
In addition, any patient with suspected phaeochromocytoma needs same day assessment. If none of these features are present, patients should be urgently assessed for end-organ damage. If present, treatment should be considered immediately before ABPM/HBPM. If absent, blood pressure should be reviewed within 7 days in clinic.

Any patient aged < 40 years should be considered for specialist assessment to exclude a secondary cause of hypertension.

How well did you know this?
1
Not at all
2
3
4
5
Perfectly
64
Q

Name 5 side effects of each hypertensive drugs?

A

ACE inhibitors-• Cough
• Hyperkalaemia,Angioedema

Bendroflumethiazide- Gout
• Hypokalaemia
• Hyponatraemia
• Impaired glucose tolerance

Calcium channel blockers
Headache
• Flushing
• Ankle oedema

Beta-blockers •
Bronchospasm (especially in asthmatics)
• Fatigue
• Cold peripheries

Doxazosin
• Postural hypotension

How well did you know this?
1
Not at all
2
3
4
5
Perfectly
65
Q

How to check for true postural hypotension

A

Orthostatic (postural) hypotension is an excessive fall in blood pressure (BP) when an upright position is assumed. The consensus definition is a drop of>20 mm Hg systolic, 10 mm Hg diastolic, or both. Symptoms of faintness, light-headedness, dizziness, confusion, or blurred vision occur within seconds to a few minutes of standing and resolve rapidly on lying down. Some patients experience falls,syncope, or even generalizedseizures. Exercise or a heavy meal may exacerbate symptoms. Most other associated symptoms and signs relate to the cause.

How well did you know this?
1
Not at all
2
3
4
5
Perfectly
66
Q

What medication can cause postural hypotension?

A

Antipyschotic

Antidepressants

Anti hypertension-ACEI,beta blocker

leva dopa

How well did you know this?
1
Not at all
2
3
4
5
Perfectly
67
Q

What is Phaecytochromia?How is to investigated and managed?

A

Phaeochromocytoma is a rare catecholamine secreting tumour. About 10% are familial and may be associated with MEN type II, neurofibromatosis and von Hippel-Lindau syndrome

Basics
bilateral in 10%
malignant in 10%
extra-adrenal in 10% (most common site = organ of Zuckerkandl, adjacent to the bifurcation of the aorta)

Features are typically episodic
hypertension (around 90% of cases, may be sustained)
headaches
palpitations
sweating
anxiety

Tests
24 hr urinary collection of metanephrines (sensitivity 97%*)
this has replaced a 24 hr urinary collection of catecholamines (sensitivity 86%)

Surgery is the definitive management. The patient must first however be stabilized with medical management:
alpha-blocker (e.g. phenoxybenzamine), given before a
beta-blocker (e.g. propranolol)

How well did you know this?
1
Not at all
2
3
4
5
Perfectly
68
Q

How can AKI be diagnosed?

A

AKI may be classified by a number of systems including RIFLE and KDIGO.
A number of different staging systems have been proposed to help grade the severity of AKI including the ‘RIFLE’ criteria, ‘AKIN’ criteria and more recently the ‘Kidney Disease: Improving Global Outcomes’ (KDIGO) criteria.

Based on the KDIGO criteria, an AKI is defined by one of the following parameters:

An increase in serum creatinine by ≥ 26.5 micromol/L within 48 hours
An increase in serum creatinine to ≥ 1.5 times baseline within 7 days
Urine output < 0.5 mL/kg/hr for six hours

How well did you know this?
1
Not at all
2
3
4
5
Perfectly
69
Q

What are features of history and clinical examination that may indicate AKI?

A
Risk factors
There are a number of risk factors that increase the likelihood of developing an AKI during hospital admission. 
Age (> 65 years old)
History of AKI
CKD
Urological history (e.g. stones)
Cardiac failure
Diabetes mellitus
Sepsis
Hypovolaemia
Nephrotoxic drug use
Contrast agents
Patients usually present with clinical features of hypovolaemia & dehydration.
Reduced capillary refill time
Dry mucous membranes
Reduced skin turgor
Thirst
Dizziness
Reduced urine output
Orthostatic hypotension 
It is important to consider features associated with fluid loss including excessive sweating, vomiting, diarrhoea and polyuria. In elderly patients, there may also be evidence of confusion. 

In patients with renal hypoperfusion in the context of hypervolaemia (e.g. cardiac failure), it is important to assess for fluid overload.

Ankle swelling
Orthopnoea
Paroxysmal nocturnal dyspnoea
Dyspnoea
Raised JVP
Ascites 
Intrinsic renal

The clinical presentation of intrinsic renal AKI is dependent on the specific aetiology.
Patients with ATN will demonstrate features consistent with the underlying aetiology. Those with intrinsic glomerular pathology may present with features of nephritic syndrome (e.g. haematuria, proteinuria, oliguria and hypertension) or nephrotic syndrome (e.g. heavy proteinuria, hypoalbuminaemia and oedema).

Patients with a tubulointerstitial disease (e.g. acute interstitial nephritis) may complain of arthralgia, rashes and fever. Eosinophilia is frequently seen.

Post-renal

The clinical features of post-renal AKI depend on the site, chronicity and laterality (unilateral or bilateral) of the obstruction.
Patients with urinary stones may present with classical loin-to-groin pain, haematuria, nausea and vomiting. Those with prostatic problems may have lower urinary tract symptoms (e.g. dysuria, frequency, terminal dribbling, hesitancy). Obstruction at the bladder neck might be associated with a palpable bladder and a tender suprapubic area.

How well did you know this?
1
Not at all
2
3
4
5
Perfectly
70
Q

How is AKI managed and What is RENAL DRS 26?

A

anagement
The management of an AKI should involve regular assessment and monitoring, controlling volume dysregulation and correcting electrolyte abnormalities and metabolic acidosis.
Principles of management

Management is guided by the underlying cause. Here we will discuss the general principles of management that can be applied to most cases of AKI.

Patients can be staged according to the KDIGO criteria. It is suggested that patients who have stage 3 AKI or a suspected diagnosis that may require specialist intervention (e.g. glomerulonephritis, systemic vasculitis), be discussed with a nephrologist within 48 hours of detection. Patients with post-renal AKI may require discussion with a urologist.

Regular assessment and monitoring

Regular assessment of the patients’ fluid status should be completed including monitoring their urine output, which may require a urinary catheter and daily weights.

A baseline creatinine should be recorded and serial U&Es taken daily, increased to twice daily in more severe cases.

Nephrotoxic drugs should be stopped (e.g. ACEi, NSAIDs, spironolactone) and regular prescriptions should be altered to reflect the change in creatinine clearance.

Volume dysregulation

If patients are hypovolaemic then intravenous fluids should be prescribed. The amount and type of fluids will depend on the clinical status of the patient.

If the patient is hypervolaemic they may require fluid restriction +/- the use of diuretics. Diuretics (e.g. furosemide) should be used carefully in renal impairment as they can be nephrotoxic.

Electrolyte abnormalities

Severe hyperkalaemia, variably defined as >6.5 or 7 mmol/L, is a medical emergency.
The management of hyperkalaemia is critical to avoid potential life-threatening arrhythmias. It involves:

Protection of the myocardium: 10ml of 10% calcium gluconate.
Reduce extracellular potassium: aim is to drive potassium into the intracellular compartment. Insulin (e.g 10 units ACTRAPID in 100ml 20% dextrose) and beta agonists (e.g. 2.5mg nebulised salbutamol) are given.
Additional: stop or adjust potassium-sparing or potassium-containing medications. Resins can reduce potassium absorption but these take hours/days to have effect.
Other electrolyte problems include hypocalcaemia and hyperphosphataemia.

Metabolic acidosis

The handling of acid-base is impaired in the setting of AKI due to a reduction in the GFR. This can results in a metabolic acidosis. Depending on the severity of acidosis and associated clinical state, choices for management involve the use of sodium bicarbonate or dialysis.

Complications

The major complications that can occur in association with AKI include hyperkalaemia, fluid overload, metabolic acidosis and uraemia.

The development of uraemic complications (e.g. encephalopathy, pericarditis), hyperkalaemia, fluid overload or metabolic acidosis that are refractory to medical therapy warrant urgent dialysis.

A useful mnemonic for assessment and management of any patient presenting with an acute kidney injury.
RENAL DRS 26 mnemonic

 R-record baseline creatinine
E-Exclude obstruction
N-Stop nephrotoxic drugs
A-Assess fluid status
L-losses and catheterisation
D-Dipstick
R-Review meds
S-Renal screen
26-Greather than 26 rise in creatinine
How well did you know this?
1
Not at all
2
3
4
5
Perfectly
71
Q

What are the causes of CKD?

A

Major causes include:

Hypertensive nephropathy
Diabetic nephropathy
Glomerulopathties
Inherited kidney disorders (e.g. PCKD)
Ischaemic nephropathy (e.g. vascular disease)
Obstructive uropathy
Tubulointerstitial diseases
Medications
72
Q

What are the signs and symptoms of CKD?

A

Clinical features
Patients are generally asymptomatic with CKD, but start to develop non-specific symptoms at more advanced stages (e.g. eGFR < 45ml/min).
It is always important to look for evidence of an underlying cause of CKD (e.g. large bilateral abdominal masses could be suggestive of PCKD).

Symptoms

Frequently asymptomatic in early stages
Anorexia & nausea
Fatigue & weakness
Muscle cramps
Pruritus
Dyspnoea
Oedema
Signs
Pallor (secondary to anaemia)
Hypertension
Fluid overload (e.g. raised JVP, peripheral & pulmonary oedema)
Skin pigmentation
Excoriation marks
Peripheral neuropathy​
73
Q

What are the Indication and contraindications for Renal biopsy?

A

Indications
Unexplained hematuria, proteinuria, AKI ,glomerular disease

Contraindications

Absolute-Clotting abnormality ,uncontrolled HTN, skin site infection
Relative-Uncoperative patient, small kidney solitary kidney,Anatomic abnormality

74
Q

How can CKD be managed and what is the criteria for starting ace

A

The principles of CKD management are to treat the underlying cause, prevent or slow progression (e.g. renoprotective therapy), treat associated complications and plan for RRT.
Management of chronic kidney disease (CKD)

Renoprotective therapy
Renoprotective therapy is aimed at slowing the progression of CKD, independent of the aetiology.
Renoprotective therapy is centered around blood pressure control and reducing proteinuria. Specific blood pressure targets depend on whether CKD is secondary to diabetes and the presence of proteinuria.

A standard BP target is < 130/80 mmHg if the patient is diabetic or has albuminuria. Therapy to control BP utilises ACE inhibitors and angiotensin receptor antagonists (both renin-angiotensin system antagonists). These drugs are both antihypertensive and antiproteinuric.

Renin-angiotensin system antagonists should be offered to patients who are:

Diabetic and have an ACR of 3 mg/mmol or more
Hypertensive and ACR of 30 mg/mmol or more
ACR > 70mg/mmol independent of CVS disease
Outside of these parameters, hypertension should be treated in accordance with the usual NICE hypertension guidance (typical target BP < 140/90 mmHg).

Other therapies to consider include:

Statin therapy
Smoking cessation
Protein restriction: now not routinely recommendated as lack of quality evidence (discussed in the renal association practice guidelines)
Antiplatelets for secondary prevention of CVS disease
Treating complications
A number of important complications develop as a consequence of CKD, which include anaemia, hyperkalaemia, mineral and bone disorders, fluid overload and acidosis.

The management of mineral and bone disorders requires management of the underlying biochemical abnormalities.

Hypocalcaemia: dietary supplements and calcitriol.
Hyperphosphataemia: dietary restriction and phosphate binders.
Hyperparathyroidism: calcimimetics or surgery.
Fluid overload

In the presence of significantly reduced GFR, the kidneys are unable to adequately controlled fluid volume. This leads to hypervolaemia and patients may have evidence of peripheral oedema, ascites, raised JVP, gallop rhythm and bilateral pleural effusions.

Fluid overload can be managed with a combination of fluid restriction, reduced sodium intake and the use of oral diuretics (e.g. furosemide).

It is still important to assess patients for other potential causes of anaemia (e.g. iron-deficiency, folate deficiency), which can subsequently be corrected. The main management for anaemia in CKD is the use of erythropoietin-stimulating agents (ESA) such as epoetin alfa.

Hyperkalaemia

The ability of the kidneys to maintain adequate acid-base homeostasis and electrolyte balance diminishes with worsening renal function. Many medications, including NSAIDs and potassium-sparing diuretics, may worsen hyperkalaemia. Furthermore, uncontrolled metabolic acidosis may also worsen potassium handling.

Acute rises in potassium should be managed as a medical emergency. This involves stabilisation of the myocardium (with calcium gluconate) and driving potassium into the intracellular compartment (with insulin/dextrose). Chronic elevations in serum potassium can be managed with low potassium diets, potassium-binding resins and correction of acidosis.

Acidosis

Patients with CKD have an increased tendency to retain hydrogen ions because of abnormalities in their acid-base homeostasis.

This leads to low bicarbonate levels and management generally involves the use of oral sodium bicarbonate therapy.

Renal replacement therapy
Haemodialysis, peritoneal dialysis and renal transplant are all forms of RRT that are indicated for ESRD.

Haemodialysis-hypotension,disequilibrium syndrome, allergy

Peritoneal- Continuous and Automated

Complications-SBP

75
Q

What is renal osteodystrophy?

A

Bone pathology associated with cad

Osteotitis fibrosa cystica
Osteomalacia
Adynamic bone disease-no bone turnover(PTH suppression)

76
Q

What are think you need to consider in ESRD?

A

1) Aetiology-HTN,DM,Glomerular disease
2) Plan and care-transplant,dialysis
3) Access -fistula,peritoneal,tunneled catheter
4) Frequency and last session
5) Units-last seen, current treatment, complications

77
Q

What is Pulmonary Hypertension and how can it be classified?

A

Recall pathology

various factors cause vascular injury leading to reduce NO and increase endothelin reducing vasodilation and increasing pressure

Defined as mean pulmonary artery pressure (mPAP) ≥ 20mmHg at rest (measured by right heart catheterisation)

Considered severe when:
mPAP ≥35 mmHg
mPAP ≥20 mmHg + elevated right atrial pressure
+/- cardiac index <2L/min/m2

Classification

Group 1: Pulmonary Arterial Hypertension (PAH) with Pulmonary Artery Occlusion Pressure <15mmHg
Idiopathic PAH
Hereditary PAH
Drug and toxin-induced PAH
PAH associated with
Connective tissue disease
HIV infection
Portal hypertension 
Congenital Heart Disease
Schistosomiasis

Group 2: Pulmonary Hypertension due to left heart disease
Heart failure with preserved ejection fraction
Heart failure with reduced ejection fraction
Valvular heart diseases

Group 3: Pulmonary Hypertension due to lung diseases/ hypoxia
Obstructive lung disease
Restrictive lung disease
Other lung disease with mixed restrictive/ obstructive pattern
Developmental lung disorders

Group 4: Pulmonary Hypertension due to pulmonary artery obstructions
Chronic thromboembolic pulmonary hypertension (CTEPH)
Other pulmonary artery obstructions
Group 5: Pulmonary Hypertension with unclear/ multifactorial mechanism
Systemic and metabolic disorders
Others
Complex congenital heart diseases

78
Q

What are the risk factors, signs and symptoms of pulmonary hypertension?

A

Risk factors

Family history of pulmonary arterial hypertension
Congenital heart disease
Connective tissue disease (systemic sclerosis, SLE)
Drugs and toxins
Aminorex, Methamphetamine, Fenfluramine
Aminorex =a type of SSRI, stimulates weight loss
Fenfluramine= appetite suppressant
Human Immunodeficiency Virus (HIV)
Mechanism not known
Portal hypertension
Genetics-Abnormal BMPR2 gene

symptoms and signs

Shortness of breath
Chest pain
Dizziness
Syncope
Fatigue
Oedema
Dry cough
Raynaud’s phenomenon 
Skin- Telangiectasias, Raynaud’s phenomena, Sclerodactyly
Increased jugular venous pressure (JVP)
Hepatojugular reflux
Peripheral oedema +/- ascites 
Heart sounds:
Accentuated split S2
Third heart sound present
Tricuspid regurgitation- heard best at left lower sternal border
79
Q

How is pulmonary Hypertension diagnosed and managed?

A

Diagnosis-Echo-right heart catheterisation

check notion for pathway
Management depend on classification

type 1-pulmonary vasodilators,prostonoids(illoprost),ccb,long term anticoagulation like wafarin

check notion

lung transplant
New York Heart Association (NYHA) functional class III
or IV
Mean right atrial pressure >10mmHg Mean pulmonary arterial pressure
>50 mmHg
Failure to improvefunctionally despite medical therapy
Rapidly progressive disease

80
Q

What are the CXR findings of primary hypertension?

A

Large central pulmonary arteries
peripheral pruning
Enlarged right heart

81
Q

What are the Indications and Contraindications for lung transplant?

A

Indications
Failure of medical and surgical improvement of disease
Less than a 2 year expected survival from disease
Progressive exercise intolerance
CO2 retention
Increased O2 requirements
Right Heart Failure

Contraindictions
Incurable Malignancy
Old Age
–	>69
Active or incurable infection
–	HIV, Hep C…
Other major organ system damage (kidney, liver)
Morbid obesity
Alcohol, smoking or drug abuse
Corticosteroid Therapy
Previous CT surgery (case by case)
82
Q

What complications can develop after a lung transplant and what follow up is needed?

A

Follow up
Lung Biopsies done at 2 weeks, 4-6 weeks and 12 weeks, 6 months, and then yearly
Acute Rejection usually happens within the first year
Occurs in 36% of transplants
Diagnosis includes:
⚫ Fever, dyspnea, impaired gas exchange (↓ PaO2), ↓
forced expiratory volume (airway flow)

complications

Infections are the leading cause of morbidity and mortality in lung transplants
Bacterial, Viral, and Fungal Infections
Infection rate in lung tx is higher than any other tx
Due to airway colonization having direct exposure to the lung allograft
⚫ And the lack of the cough relfex
Bacterial infections often originate from the donor lungs
Recipient can provide their own infections pre-op, common in Cystic Fibrosis patients

Acute
Bleeding
acute rejection
REPERFUSION INJURY
electrolyte abnormalities
arrhythmia
kidney problems
Chronic
chronic rejection
skin cancers
DM
HTN
Arrthymias
CKDA
83
Q

When is a heart lung transplant done and what are the complications?

A

Restricted to patients with: (mostly)
Congenital heart disease’s with Eisenmenger’s Syndrome (#1 indication for HLT)- 35%
⚫ Trunchus arteriosus, pulmonary atresia, HLHS
Idiopathic pulmonary arterial HTN- 25%
Cystic Fibrosis- 14%

Complications
HTN- 88%%
Renal Dysfunction- 28%
Hyperlipidemia- 66%
Diabetes- 21%
Bronchiolitis Obliterans- 27%
Coronary Artery vasculopathy- 8%
Survival Rates:
72% at 3 months
64% at 1 year
84
Q

What are the clinical features od chronic beryllium disease?and how it diagnosed?

A

Manufacturing of fluorescent light tubes, ceramics, electronics, aerospace industries etc
Workers at risk:
Processes generating airborne beryllium particles

Acute
Chronic-
Granulomatous inflammatory disorder
Very similar to sarcoidosis

Clinical features
Insidious onset
Symptoms
Dyspnoea on exertion
Cough
Fatigue
Crackles upon auscultation 
Anorexia and weight loss
Fever
Chest pain

Lab
History of beryllium exposure
Positive lymphocyte proliferation test (LPT)
Peripheral blood/ bronchoalveolar lavage
LPT to confirm sensitisation to beryllium
Transbronchial biopsy of lung tissue to confirm presence of disease
Presence of epithelioid granulomas & mononuclear infiltrates
in absence of infections in lung tissues

85
Q

Who’s at risk of silicosis?What is the disease pattern?

A

Inhalation of silicon dioxide (silica)
Silica major component of rock and sand
Key elements:
History of silica exposure sufficient to cause degree of illness, with appropriate latency period from time of first exposure
Absence of another diagnosis more likely to be responsible for observed abnormalities

Acute silicosis
Intense exposure to fine dust of high silica content over several months
Subacute/ Accelerated silicosis
Shorter duration, heavier exposures (2-5 years) compared to chronic simple silicosis
Chronic simple silicosis
Most common form
>10 years exposure to dust <30% quartz (Silicone dioxide SiO2)

CXR findings

Hallmark:
Silicotic nodules/ islet in pulmonary parenchyma and hilar lymph nodes
Nodules appear as small, round opacities (<10mm) in bilateral upper lung zones on chest x-ray
Calcification of lesions forming “egg-shell” appearance
Upper lobes fibrosis

Signs and symptoms

Few symptoms and signs
Pulmonary function test
Usually normal
Occasionally mild restrictive and decreased lung compliance
Disease progresses to complicated silicosis as fibrosis progressively develops in upper lobes

86
Q

What is Progressive massive fibrosis?

A

Small, silicotic nodules coalesce into larger, fibrotic masses
Tend to occur in upper lung fields
May obliterate blood vessels and bronchioles
Distort lung architecture
Leads to respiratory insufficiency

87
Q

What is pneumoconiosis?What is the Pathophysiology?

A

Lung tissue reaction to presence of
dust accumulation in lungs

Essential factors for clinical pneumoconiosis:
Exposure to specific substance:
Silica and asbestos: Potent biological effect
Coal: May accumulate to considerable amounts, but minimal tissue response
Size of particles between 1-5 μm
Commonly retained in lungs
Exposure for sufficient length of time
~10 years

88
Q

What are the complication of Silicosis and how is it managed?

A

Complications

Susceptible to mycobacterial and fungal diseases (e.g. cryptococcosis)
Silicotuberculosis
Silica is cytotoxic to alveolar macrophages
Patients at increased risk of developing TB
Autoimmune disorder:
Rheumatoid arthritis
Scleroderma
Malignancy

Management

No definitive treatment
Most important aspect: Prevention from silica exposure in work environment
Advice change of occupation to avoid further silica dust exposure
Advice smoking cessation
Control and prevent development of TB in patients
Tuberculin skin test/ interferon-γ assay test to identify TB status
Lung lavage to remove silica from alveoli
Consider lung transplant in acute and accelerated silicosis

89
Q

What is Caplans syndrome?

A

Rheumatoid arthritis + pneumoconiosis
Manifest as intrapulmonary nodules
Appear homogenous and well-defined on chest X-ray
Lesions grow rapidly, cavitate, and might produce pneumothorax
Compared to silicosis/ CWP where nodules appear over a period of time

90
Q

How can Asbestosis affect the lungs?and what are the x-ray finding?

A

Asbestos widely used to construct ceiling, walls and flooring of houses/ buildings
Provide insulation and fire protection
Latency period
Several years before pleural \Very high incidence before 1980s
Asbestos widely used to construct ceiling, walls and flooring of houses/ buildings
Provide insulation and fire protection
Latency period
Several years before pleural thickening develops
>20 years for fibrosis & plaque

Parenchymal asbestosis

Diffuse interstitial fibrosis with restrictive pattern on pulmonary function test
Impaired gas exchange in alveoli
Presents with progressive exertional dyspnoea and fatigue
Takes >10 years for radiographic changes to develop

Asbestos related pleural abnormalities
Types of abnormalities:

Pleural plaques
Benign asbestos pleural effusion
Diffuse pleural thickening
Rounded atelectasis
Mostly asymptomatic
Some patients may develop progressive dyspnoea, intermittent chest pain, cough
CXR
Small, irregular, oval opacities
Diffuse interstitial fibrosis
“Shaggy heart border” sign 
Cardiac silhouette ill-defined on chest radiograph
Implies pleural
91
Q

What is Mesothelioma?How is it diagnosed and managed?

A

Malignancy/ thickening at pleural lining of lungs
Usually linked to chronic asbestos exposure
Symptoms:
Normally asymptomatic until advance stages
Dyspnoea, chest pain, fatigue

CXR
Mass at mid zone of left lung (pushes trachea towards right)

Pleural thickening
(Increased radiopacity at pleura linings )

Pleural effusion (Meniscus sign: blunted costophrenic angle

92
Q

What are the risk factors for OSA?

A
Obesity (Strongest Risk Factor)
Male
Middle age (30-50 years old)
Hypothyroidism
Acromegaly
Use of sedatives/ narcotics/ alcohol
Smoking
93
Q

What are the complications for OSA?

A
Increased risk of vascular diseases:
Systemic hypertension
Pulmonary hypertension (due to hypercapnia)
Arrhythmia
Nocturnal bradycardia during apnoea, tachycardia upon resolution
Myocardial infarction
Heart failure
Stroke
Sudden death

Excessive daytime sleepiness
Reduced concentration in daily activities
Road-traffic accidents due to driver sleepiness (risk 7x greater)

Repetitive hypoxemia and hypercapnia
Reduced airflow into lungs
Subsequent bodily oxygen desaturation
Chemoreflex mediated sympathetic activation
Increased sympathetic nerve activity + increased catecholamine
Increased cardiac output & vasoconstriction
BP could rise to 220/130mmHg during apnoea
Diastolic nocturnal hypertension
Loss of nocturnal dipping of blood pressure

OSA Cardinal Symptoms (3S):
Snoring
Sleepiness
Sleep Apnoea Episodes

94
Q

What investigations are done for OSA?

A

Epworth sleep scale
Self-administered 8-question questionnaire
Respondents asked to rate from 0-3 on their usual chance of dozing off while doing activities listed on questionnaire
Scores:
0-10: normal daytime sleepiness
11-12: mild excessive daytime sleepiness
13-15: moderate excessive daytime sleepiness
16-24: severe excessive daytime sleepiness

Examination
Inspection
Obesity
Micrognathia/ retrognathia (small, receding lower jaws)
Macroglossia (enlarged tongue)
Neck size >17 inches
Nasal congestion/ nasal polyps
Tonsillar hypertrophy
Adenoid enlargement 

Bedside

Blood pressure
Assess systemic hypertension (complications of OSA)

ECG
Rule out arrhythmias, myocardial infarction etc (complications of OSA)
Arterial blood gas
Look for pO2 and pCO2 (patient with OSA experience hypoxemia and hypercapnia)

Bloods
Full blood count
Patient might have polycythaemia secondary to chronic hypoxemia
Thyroid function test
Hypothyroidism- risk factor of OSA 

Imaging
Echocardiography
Assess presence of left ventricular hypertrophy (complication of OSA)
CT/MRI
Cephalometric analysis
Look at structure of airways, adenoids, tongue etc

Diagnosis made through overnight sleep study(Polysonography)
Overnight sleep study of varying complexity could be performed
Depending on patient preference and local arrangements
Inpatients
At home

95
Q

What are the signs of acute limb ischemia and how is it managed?

A

Acute limb ischaemia is defined as severe, symptomatic hypoperfusion of a limb occurring for <2 weeks. Even though the definition talks of 2 weeks, it is a surgical emergency and needs to be corrected as soon as possible, ideally within 4-6 hours.

The 6Ps of acute limb ischaemia

Pulseless
Painful
Pale
Paralysis
Paraesthesia
Perishingly cold
In real life if the limb has lost motor and sensory function then it is almost certaintly unsalvageable so these signs are not particularly useful if you want to try and save the limb!

Management

As in all emergencies, approach using the DR ABCDE algorithm:

As part of B, administer oxygen.
As part of C, administer intravenous fluids and analgesia. Take bloods for FBC, U&E, group and save, and clotting. ECG is important to see whether the patient is in atrial fibrillation (this suggests an embolic cause).
The next steps are to urgently inform vascular surgery.

The patient should be kept nil by mouth in preparation for surgery.
Intravenous heparin (to prevent thrombus propagation) may be administered (typically after senior review).
Definitive management depends on whether there is complete or incomplete limb ischaemia, and whether the cause is thrombotic or embolic.

Management of thrombotic causes

In thrombotic causes:

If ischaemia is incomplete (and the limb is likely to remain viable for 12-24 hours), patients should have angiography to map the occlusion site and plan intervention. Endovascular procedures may be an option (such as angioplasty, thrombectomy, or intra-arterial thrombolysis).
If ischaemia is complete patients must be urgently taken to theatre for bypass surgery. Angiography and thrombolysis will delay management.
Management of embolic causes

In embolic causes: the leg is typically threatened and immediate embolectomy is required. Angiography can be used to confirm the adequacy of the procedure. If embolectomy fails, on-table thrombolysis may be considered.
Note that if the limb is non-viable amputation may be required.

96
Q

What is the first line investigation for PAD?

A

ny patient suspected or diagnosed as having PAD should have a full cardiovascular risk assessment including blood pressure, FBC, blood glucose, lipids and electrocardiogram.

Other specific investigations include:

Ankle-Brachial Pressure Index (ABPI) is a simple, first-line investigation for peripheral artery disease (PVD) that can be performed on almost all patients. It is performed by using a doppler probe for find the systolic brachial blood pressures of the arms and comparing them to the ankle blood pressures in the feet. The ABPI is calculated as follows:

Brachial pressure (on side of interest)/Ankle pressure (on side of interest)

Interpretation of ABPI

Interpretation is as follows:

More than 1.2: abnormal thickening of vascular walls (think diabetes)
0.9 - 1.2: Normal
0.8 - 0.9: Mild disease
0.5 - 0.8: Moderate disease
Less than 0.5: Severe disease
Imaging investigations

A normal ABPI in a diabetic patient CANNOT exclude peripheral vascular disease and they will almost certainly need further investigation.

First line-Duplex arterial ultrasound - for those who might be suitable for revascularisation.
Can determine the site, severity and length of stenosis.
MR arteriogram - used for those who are candidates for revascularisation
CT arteriogram - used in those unsuitable for MR
Digital subtraction angiography - usually performed at the time of intervention or for monitoring disease

97
Q

How can PAD be treated?

A

Management of the chronically ischaemic leg (non-surgical)

Conservative measures include risk factor modification. This involves offering referral for a supervised exercise programme. Advise should be given on smoking cessation and weight management.

Medical management involves:

A. Managing cardiovascular risk:

Antiplatelet therapy: with clopidogrel 75mg once daily. Aspirin is prescribed only if clopidogrel is not tolerated or contraindicated.
Lipid lowering therapy: with atorvastatin 80mg once nightly.
In diabetics, glycaemic control should be optimised.
High blood pressure should be managed appropriately.
B. Managing pain with appropriate analgesia:

Naftidrofuryl oxalate is a vasodilator which can alleviate pain in peripheral vascular disease. It should only be prescribed if supervised exercise is ineffective and the patient does not want to be referred for angioplasty or bypass surgery.
Management of the chronically ischaemic leg (surgical)

Patients with intermittent claudication should be referred for endovascular revascularisation or surgical revascularisation when risk factor modification has been introduced and supervised exercise programme has not led to symptom improvement.

For critical limb ischaemia (where there is rest pain, tissue loss, and ankle artery pressure of <50 mmHg) patients should be urgently referred to the vascular multidisciplinary team for endovascular revascularisation or surgical revascularisation.

Endovascular methods are recommended for small discrete stenosis.
Surgical bypass is recommended for larger more extensive stenosis.
Amputation may be required if there is: critical limb ischaemia unsuitable for other interventions, intractable pain, an unresolving ulcer, or severe loss of function.

98
Q

What are the causes or risk factors for aneurysm

A
Hypertension
Atherosclerosis-Atherosclerotic ulcer
Genetical-Marfan's,Ley's Dietz,Ehlers-Danlos,Biscuspid aortic valve,Herditary thoracic AAA
Coarctation of aORTA
Turner's
TOF
Trauma-stent.ect
Pregnancy
Inflammatory-Takayasu
99
Q

What are the criteria for surgery and screening

A

Most aneurysms are asymptomatic. They can present with abdominal mass (which is usually pulsatile and expansile)

Screening and repeat screening principles

In the UK, screening is offered at age 65 using abdominal ultrasound scan.

If small AAA (3-4.4cm) – offered yearly repeat ultrasound
If medium AAA (4.5-5.4cm) – offered repeat ultrasound every 3 months
If large AAA (>5.5cm) – surgery generally recommended.
Management

Surgical options: The two main surgical options are open repair or Endovascular Aneurysm repair (EVAR).

The indications for repair are size >5.5cm or rapid expansion.

Rapid expansion is defined as increase in diameter >5mm over a 6 month period or >10mm over one year. These limits have been developed due to clinical evidence which balances the risk of surgery vs the risk of aneurysm rupture. It is very uncommon for aneurysms below 5cm to rupture.

100
Q

How does dissection present and what complications can occur?

A
Associations
hypertension: the most important risk factor
trauma
bicuspid aortic valve
collagens: Marfan's syndrome, Ehlers-Danlos syndrome
Turner's and Noonan's syndrome
pregnancy
syphilis
Amphetamine use

Features:
chest pain: typically severe, radiates through to the back and ‘tearing’ in nature
pulse deficit
weak or absent carotid, brachial, or femoral pulse
variation (>20 mmHg) in systolic blood pressure between the arms(radio-radial and radio femoral delay)CT angiogram is used to diagnose dissection but other investigations can suggest the diagnosis and/or its complications:

Complications
Complications of backward tear
aortic incompetence/regurgitation
MI: inferior pattern is often seen due to right coronary involvement

Complications of a forward tear
unequal arm pulses and BP
stroke
renal failure

Death due to internal haemorrhage
Rupture

Cardiac tamponade
Stroke
Limb ischaemia
Mesenteric ischaemia

101
Q

How are aortic dissections classified and treated?

A

Inx
ECG - May show ischaemia in specific territories if dissection extends into coronary arteries.
Echocardiogram - May demonstrate pericardial effusion and aortic valve involvement.
Chest x-ray - May show a widened mediastinum
Bloods:
Troponin may be raised
D-dimer may be positive
Prognosis

Prompt diagnosis and treatment is required as rupture carries an 80% mortality rate.

Initial management

Resuscitation if necessary
Cardiac monitoring
Strict blood pressure control (e.g. IV metoprolol infusion)
Definitive management

Depends on the type of dissection

Type A: Usually requires surgical management (e.g. aortic graft)
Type B: Normally managed conservatively with blood pressure control. If there is evidence of end organ damage then endovascular/open repair may be performed.

Stanford classification
type A - ascending aorta, 2/3 of cases
type B - descending aorta, distal to left subclavian origin, 1/3 of cases

DeBakey classification
type I - originates in ascending aorta, propagates to at least the aortic arch and possibly beyond it distally
type II - originates in and is confined to the ascending aorta
type III - originates in descending aorta, rarely extends proximally but will extend distally

Type A
surgical management, but blood pressure should be controlled to a target systolic of 100-120 mmHg whilst awaiting intervention

Type B*
conservative management
bed rest
reduce blood pressure IV labetalol to prevent progression

102
Q

What is Beurger’s Disease?How to treat it?

A

Definition

Buerger’s disease is a non-atherosclerotic vasculitis caused by occlusion in small and medium-sized arteries.

Epidemiology

It is most common in young male smokers of Mediterranean and Middle Eastern origin.

Clinical Features

It typically presents as an acutely ischaemic limb, without a background of peripheral claudication.

Investigations

Investigations are aimed at excluding differentials (such as atherosclerotic disease and autoimmune disease). Arterial Doppler will confirm the absence of peripheral pulses in the affected limb. Further imaging (such as with arterial duplex or CT/MR angiography) will show non-atherosclerotic occlusion. Martorell’s sign on arterial duplex describes the ‘corkscrew’-shaped collateral vessels characteristic of Buerger’s disease.

Management

Management is with smoking cessation ± vasoactive medication (such as nifedipine).

If there is critical ischaemia the patient requires hospital admission, vasoactive medication and debridement of gangrenous tissue.

103
Q

What are the causes of Raynaud’s?

A

aynaud’s phenomenon is characterised by an exaggerated vasoconstrictive response of the digital arteries and cutaneous arteriole to the cold or emotional stress. It may be primary (Raynaud’s disease) or secondary (Raynaud’s phenomenon).

Raynaud’s disease typically presents in young women (e.g. 30 years old) with bilateral symptoms.

Secondary causes of Raynaud's phenomenon
connective tissue disorders
scleroderma (most common)
rheumatoid arthritis
systemic lupus erythematosus
leukaemia
type I cryoglobulinaemia, cold agglutinins
use of vibrating tools
drugs: oral contraceptive pill, ergot
cervical rib
Factors suggesting underlying connective tissue disease
onset after 40 years
unilateral symptoms
rashes
presence of autoantibodies
features which may suggest rheumatoid arthritis or SLE, for example arthritis or recurrent miscarriages
digital ulcers, calcinosis
very rarely: chilblains

Management
all patients with suspected secondary Raynaud’s phenomenon should be referred to secondary care
first-line: calcium channel blockers e.g. nifedipine
IV prostacyclin (epoprostenol) infusions: effects may last several weeks/months

104
Q

How does Marfan’s Syndrome present?

A

Marfan’s syndrome is an autosomal dominant connective tissue disorder. It is caused by a defect in the FBN1 gene on chromosome 15 that codes for the protein fibrillin-1. It affects around 1 in 3,000 people.

Features
tall stature with arm span to height ratio > 1.05
high-arched palate
arachnodactyly
pectus excavatum
pes planus
scoliosis of > 20 degrees
heart: dilation of the aortic sinuses (seen in 90%) which may lead to aortic aneurysm, aortic dissection, aortic regurgitation, mitral valve prolapse (75%),
lungs: repeated pneumothoraces
eyes: upwards lens dislocation (superotemporal ectopia lentis), blue sclera, myopia
dural ectasia (ballooning of the dural sac at the lumbosacral level)

The life expectancy of patients used to be around 40-50 years. With the advent of regular echocardiography monitoring and beta-blocker/ACE-inhibitor therapy this has improved significantly over recent years. Aortic dissection and other cardiovascular problems remain the leading cause of death however.

105
Q

What Conditions can cause Diffuse neuronal damage?

A

Diabetic-sensory

drugs-sensory

Alcohol-sensory

CMT-mixed(Charcot-Marie-Tooth disease)

CIDP-motor

Chronic inflammatory demyelinating polyradiculoneuropathy (CIDP) is a rare type of autoimmune disorder. In an autoimmune disease, the body attacks its own tissues. In CIDP, the body attacks the myelin sheaths.

If in doubt use Surgical sieve-Infection,Inflammation,Neoplasm,Vascula,trauma/structural,Neurodegenerative,hereditary,metabolic

106
Q

What does nerve stimulation tests show in NMJ disorders?

A

Repetitive nerve stimulation show decrement in NMJ disorders like myasthenia gravis

107
Q

How can tremors be managed?

A

Conservative: relaxation techniques, reduce caffeine

Medical: propanolol, primidone

Surgical: deep brain stimulation (only for very severe/refractory cases)

108
Q

What is Tarditive dyskinesia?How is it caused and treated?

A

The most common cause of drug-induced movement disorders are dopamine receptor blocking agents. Antipsychotics, metoclopramide and prochlorperazine are prevalent examples that you will undoubtedly prescribe during your career. These medications can cause various movement disorders including tremor, neuroleptic malignant syndrome (covered in the next section), dystonia and parkinsonism.

The exact pathophysiology of tardive dyskinesia in unclear. A common theory is that there is a dopamine hypersensitivity in the nigrostriatal pathway after chronic pharmacological blockage

Treatment can be difficult and therefore prevention is crucial. Dopamine receptor blocking medications should be used in the lowest dose for the shortest time possible. Treatment involves withdrawing the offending agent and this may require an analysis of benefit vs. harm (e.g. if the antipsychotic is treating schizophrenia). There is usually a delay before improvement and changes can be irreversible. Tetrabenazine can be effective in some people who have not responded to treatment withdrawal.

109
Q

What is NMS?How is it diagnosed and treated?

A

Dopamine receptor blocking drugs can cause a severe neurological emergency called neuroleptic malignant syndrome (NMS). This can also be induced by sudden cessation of dopaminergic medications e.g. levodopa.

NMS is a clinical diagnosis, supported by the diagnostic criteria below. NMS is characterised byhigh temperatures, rigidity,autonomic dysfunction and altered mental status (often agitated confusion). Patients may demonstrate hyporeflexia.

A common scenario for this to occur would be with a patient already on an antipsychotic e.g. haloperidol (highest risk) who is then given a dopamine blocking antiemetic such as metoclopramide. The high temperature and altered mental state would make you think about infection but is important to take note of the patient’s medications.Rigidity, a new tremor and a raised CKshould make you think about NMS.

The pathophysiology of NMS is thought to be due to the abrupt loss of dopaminergic activity in the nigrostriatal pathway and basal ganglia, with fever thought to be due to dopamine blockade in the hypothalamus (which plays a role in thermoregulation).

Diagnosis-all there major criteria and at least 2 minor criteria
Major-Exposure to dopaminergic drug, rigidity,hyperthermia
minor-tachycardia,incotience,raised bp,CK

Treatment approach:

  • Stop the dopamine blocking medication
  • Supportive care: IV fluids, correct metabolic abnormalities
  • Medications in severe cases: bromocriptine (a dopamine agonist, to increase dopaminergic activity) and dantrolene (muscle relaxant to reduce rigidity, acts by inhibiting inhibiting calcium release from the sarcoplasmic reticulum in muscle cells.)
110
Q

What is serotonin syndrome?

A

Serotonin syndrome is another drug-induced syndrome that causes hyperthermia and neurological changes. In patients with polypharmacy you may need to differentiate this from NMS.

Serotonin syndrome can be caused by prescription of multiple serotonergic drugs or medication overdose. This essentially leads to serotonin toxicity and hyperstimulation in the central nervous system.

Serotonin syndrome can be caused by multiple classes of drugs that affect signaling in presynaptic and postsynaptic serotonergic neurons (in the raphe nuclei of the brainstem). The picture below demonstrates a serotonin synapse and how medications affect this.

Approach to treatment:

  • Withdraw serotonergic agents
  • Supportive care: IV fluids, cooling
  • Medications in severe cases: Benzodiazepines can be used to reduce agitation and myoclonus. Cyproheptadine (a serotonin antagonist) is sometimes used but the evidence base for this is not strongly established.We can see that a lot of symptoms overlap with NMS.
111
Q

How can Serotonin syndrome be differentiated from NMS?

A

We can see that a lot of symptoms overlap with NMS. Differentiating between the two conditions clearly begins with checking the patient’s medications and any recent changes. If a patient is pharmacologically at risk of both, their neurological symptoms are most helpful in distinguishing between the two.

Serotonin syndrome
More acute (<24hours)
Serotonergic
Increased: hyperreflexia, clonus, tremor
Diarrhoea, increased bowel sounds
NMS
More gradual (days-weeks)
Dopaminergic
Decreased: rigidity, hyporeflexia
Usually normal
112
Q

What can cause Dystonia and how can it be managed?

A

Dystonia is caused by sustained or intermittent muscle contractions / spasms that often lead the patient to hold abnormal postures or positions. This frequently causes discomfort and pain for the patient.Dystonia can have a genetic cause or be acquired from mechanisms such as trauma, stroke and drug toxicity. We will look at drug induced hyperkinesia later in this tutorial.

  • Focal:one body part affected
  • Segmental:adjacent body parts affected
  • General:entire body affected

Approach to management:

  • Conservative:education, relaxation, massage
  • Medical:A small % of patients respond to dopaminergic medication (e.g. levodopa). Benzodiazepines are also used.
  • Surgical:BoTox injection, especially useful in focal dystonia. Deep brain stimulation may be used in severe cases.
113
Q

How is Huntington’s caused and inherited?

A

The huntingtin gene (on the short arm of chromosome 4) codes for the huntingtin protein. The specific pathology of the disease is not completely characterised but mutated huntingtin protein aggregates are found in the basal ganglia, specifically the dorsal striatum (caudate and putamen), and these areas show increased neuronal death rates. There is thought to be a toxic gain-of-function in the mutant huntingtin protein.

A section of the genetic code for this protein has something called a trinucleotide repeat expansion. This is a repeating sequence of three nucleotides. In the case of huntingtin, this is a CAG repeat (which encodes glutamine and hence is known as a polyglutamine disease).

Patients with 40 or more repeats will develop the disease (100% penetrance) but patients with 36-39 repeats show a reduced penetrance. 27-35 repeats are classified as in the ‘intermediate CAG range’ and less than this is normal.For example, if a patient has a repeat number of 40, they are certain to develop the disease (assuming they survive until disease onset). A patient with 36 repeats may or may not develop HD and a patient with fewer repeats than this will not
There is a juvenile form, also known as the Westphal variant HD, due to a very large repeat count.

114
Q
  • How can Huntingtons be diagnosed and treated?
A

Howtoconfirmthediagnosis?

ForHD,atrinucleotidecytosineadenosine-guanine(CAG)equaltoorgreaterthan36repeatsisdiagnosticinthepresenceofcharacteris ticsymptoms.

Caudate and putamen atrophy

Nodisease modifyingtherapiesare currently availableforHD on the NHS. However, there is promising research, which you can read about in your “Future Treatment - Antisense Oligonucleotides” tutorial, that has the potential to drastically change our management of the disease.

Current treatments include:

  • Conservative:Symptomatic / supportivetherapy toalleviatethesymptoms
  • Medicine:Anti-choreicmedication–atypicalantipsychotics,tetrabenazine
  • Surgical:Deep brain stimulation for pharmacological resistant chorea with significant disability

Ultimately, there is no cure for HD at present and treatment revolves around supportive care (such as modifications in the patient’s house by occupational therapists, physiotherapy, speech and language therapy and additional carers) and symptomatic relief. Tetrabenazine is approved for the treatment of chorea in HD. Antipsychotics can be used to treat psychiatric symptoms, as well as often also helping with chorea

115
Q

What are the clinical features of Huntingtons?

A

3 stages-Early,middle,lates
Theinitialsymptomsoccurmorefrequentlybetweentheagesof30and50,althoughonset canrangefromchildhood/adolescence(juvenileform,alsoknownastheWestphalvariant)toindividualsolderthan70yearsofage.

HDcanpresenttotheclinicianwithoneofthreesymptomcomplexesclassicallydescribedinthiscondition(motor/physical,cognitive,andneuropsychiatric/emotional)orincombination.Themotorsymptomsincludechorea,dystonia,andtics.
Motor
Cognitive-Usually middle to late.Diffculty planning ,thinking and performing complicated tasks

Behavioural-Irritabilty ,hypersexuality ,inhibition ,depression ,auditory hallucinations,apathetic towards the late stages

116
Q

How is caudal equine managed?

A

acute surgical decompression of his L4/5 disc prolapse causing Incomplete Cauda Equina Syndrome. If surgery is required, this should happen within the next 48 hours. Possible surgeries would be a laminectomy or discetomy. The surgeons may recommend the use of high dose steroids to reduced swelling and oedema around the site of nerve compression.

The steps to take on the admissions ward are:

  • Adequate analgesia for pain control
  • Blood tests (including Group and Save and Coagulation) to prepare for surgery
  • Urethral catheter if in urinary retention or difficulty passing urine
  • Check if patient needs to be Nil By Mouth for surgery

Monitoring for deterioration or development of urinary retention.

117
Q

What are the complications of Cauda equina syndrome?

A

Patients may suffer long-term complications from Cauda Equina Syndrome, the risks of which are increased if they do not receive early and appropriate intervention. Surgery should happen within 48 hours of presentation if it is required. Patients may experience long-term issues with pain, reduced motor function, bladder/bowel disturbance and sexual dysfunction. Complete Cauda Equina Syndrome (with urinary retention) has worse long-term outcomes than Incomplete Cauda Equina Syndrome. Anyone with Incomplete Cauda Equina Syndrome should be carefully monitored whilst awaiting surgery for any developing/changing signs.

Litigation for missed/delayed management of Cauda Equina Syndrome is common. Always ask your red flag symptoms, perform thorough examination, safety net the patient, and document your findings/discussion clearly!

118
Q

What can cause bilateral leg weakness acutely?

A

Guillain-Barrésyndrome

Cauda-Equina syndrome

Stroke (if unilateral, worth keeping in mind)

Transverse myelitis

Epidural abscess

‘Off legs’ (general reduced mobility)

119
Q

How does antisense oligonucleotides work and how can these be used in treatment?

A

Recent advances in genetic medicine have allowed for the development of new treatments for single gene disorders. This is in the form of a new class of drugs called anti-sense oligonucleotides (ASOs). These compounds bind to specific mRNAs and ultimately affect protein expression in various desired ways, dependent on the disease we are treating.

Expanding on this video, the SMN protein is particularly important in promoting the survival of alpha motor neurons, which exit through the ventral roots of the spinal cord to directly innervate skeletal muscle and initiate contraction. In SMA theSMN1gene is mutated or deleted homozygously in a way that ultimately results in insufficient levels of SMN protein.

TheSMN2gene is a pseudogene copy ofSMN1,which means that it is a mutated copy which produces little or no functional protein. In this case very little functional protein is produced. This is because a single nucleotide mutation in exon 7 ofSMN2leads to it being removed in the majority of mRNAs. This resultant protein is non-functional / rapidly degraded.

Nusinersen is an ASO which binds to a complimentary sequence after exon 7 and causes mRNA processing to include this exon. Thus, functional SMN protein is produced. For many patients this leads to significant motor function and survival benefits.

Expanding on this video, the SMN protein is particularly important in promoting the survival of alpha motor neurons, which exit through the ventral roots of the spinal cord to directly innervate skeletal muscle and initiate contraction. In SMA theSMN1gene is mutated or deleted homozygously in a way that ultimately results in insufficient levels of SMN protein.

TheSMN2gene is a pseudogene copy ofSMN1,which means that it is a mutated copy which produces little or no functional protein. In this case very little functional protein is produced. This is because a single nucleotide mutation in exon 7 ofSMN2leads to it being removed in the majority of mRNAs. This resultant protein is non-functional / rapidly degraded.

Nusinersen is an ASO which binds to a complimentary sequence after exon 7 and causes mRNA processing to include this exon. Thus, functional SMN protein is produced. For many patients this leads to significant motor function and survival benefits.

So far, the most promising research has been undertaken with ASOs in patients with mutant SOD1 genes. Multiple point mutations in this gene have been found to be associated with FALS. However, the mechanism by which these mutations cause ALS is poorly characterised. Despite this results have been promising.

At the time of writing there are ongoing clinical trials for SOD1 ASOs. As with Huntington’s, the purpose of the ASO is to target mutant mRNA for degradation.

120
Q

How to diagnose and treat BPPV?

A

You can diagnose andcurethis through a couple of manoeuvres:

  • Dix-Hallpike manœuvre is used to diagnose this (Diagnose andDix-Hallpike both begin withD)
  • Eply manoeuvre can be used to knock the stones out of the semilunar canals to move to otolith out the semicircular canals

Thenystagmusin this case is torsional/rotational, which furthers point you to the patient having an inner ear cause for their dizziness. You can see this in the video above.

Once diagnosed, you can do the Epley manoeuvre, which is essentially a series of movements designed to knock the otoliths out. Obviously you don’t need to know these movements!

121
Q

What is Labyrinthitis and Vestibular neuronitis?What are the symptoms?

A

This is an inflammation of the peripheral vestibular system, commonly it follows a viral upper respiratory tract infection (like a cold!)

  • Vestibular neuronitis:Vertigo (much more common)
  • Labyrinthitis:Vertigo AND hearing loss

Unlike BPPV the vertigo is constant rather than lasting a few seconds triggered by heading movements (although head turning usually does make it feel worse!). Onset is acute and usually lasts for around a week. The patient often feels awful, very nauseous/vomiting and not keen to put up with your head-spinning neurological examination.

Always consider posterior circulation stroke or other acute neurological event as a differential for vertigo. Nystagmus can provide a good clue.

122
Q

What is Meniere’s disease?

A

This is ararecondition caused by increased production of fluid in the inner ear. It’s characterised by recurrent attacks of vertigo (lasting minutes-hours)and a feeling of aural fullness.The condition is associated with a fluctuating, often progressive sensorineural hearing loss and tinnitus.

Meniere’s disease is probably over diagnosed in the population. Vertigo is symptomatically managed with betahistine, but this is only a treatment for Meniere’s diseasenot all vertigo.

123
Q

what can affect proprioception?

A

Proprioception is the most important sensation for ensuring that we have good balance - you need to know where you are in space to move properly and keep yourself upright. Failing this, you then have to rely on your visual system to orientate you instead.

This directly reflects in the presentation of someone who has damage to their proprioception pathways. They will be profoundly unsteady, will have a broad based,ataxicgait, and will be staring at their feet.

The result is largely the same depending on where the lesion is:

  • Sensory neurones - alcohol, diabetes, Guillian-Barre
  • Dorsal root ganglion - Sjogren’s, paraneoplastic syndrome
  • Dorsal column - Tabes dorsalis (syphilis), B12 deficiency
124
Q

What can cause a postive Romberg sign?

A

Romberg’s sign is when someone becomes more unbalanced and may even fall over when their eyes are closed. This reflects that you need at least two of the three systems functioning to maintain balance:

  • Vestibular system
  • Proprioception
  • Visual system

If you know the patient’s vestibular system and visual system are working normally (which can be confirmed by examination), then you can localise the faulty system to proprioception. This can be due to an issue with peripheral sensory proprioception or the dorsal column (where the proprioception nerves travel along). Importantly it isnegative(no change to dizziness level with eyes open/closed)**in cerebellar causes of ataxia.

125
Q

What are the signs of cerebellar dysfunction?

A

If patients have an ataxic gait, but the examination of their sensory system is unremarkable, then the root of their problem could be cerebellar. The gait is not particularly different than the ataxic gait seen in sensory dysfunction.

As the cerebellum’s usual function is to contribute to co-ordination, precision and accurate timing of movement, this is reflected by the movements seen in someone who’s cerebellum is damaged. Note itdoes notinitiate movements (that’s done by the motor cortex in the brain).

There are some other signs seen typically in cerebellar dysfunction, as well as the ataxic gait. You may be familiar with the acronym DANISH:

  • D -/dysmetriadysdiadochokinesia
  • A - ataxia
  • N - nystagmus
  • I - intention tremor
  • S - scanning dysarthria
  • H - heel shin test dysfunction

There are several complicated words/signs in this acronym! The nystagmus is distinct from vestibular induced nystagmus in that the fast phase changes depending on which direction you’re looking in. An intention tremor is one that’s exacerbated by movement and gets worse as the patient moves.

So really, cerebellar dysfunction is relatively easy to spot!

. Unlike the cerebral cortex the cerebellum gets sensory information from the same side of the body it is on, and outputs to that side. In one sided damage to the cerebellum, then the symptoms will be on the same side of the body.

126
Q

What are the signs and symptoms of multiple sclerosis?

A

Triad-Optic neuritis,dystharia ,intention tremor

Fatigue:The degree of fatigue in MS is can be overwhelming and disabling

  • Cooling, Pacing activities, CBT, Mindfullness, Amantadine

Mood:Depression is common in MS

  • CBT, SSRIs, Duloxetine

Cognition:Cognition deteriorates as disease becomes advanced

  • Social support, rule out sleep issues/pain/depression

Spasticity:This can lead to pain and discomfort

  • Physiotherapy, Baclofen, BoTox

Pain:Typically neuropathic type pain

  • CBT, Amitriptyline, Gabapentin, Pregabalin

Neurogenic bladder issues:Urinary frequency, urgency, nocturia. Frequent UTIs.

  • Fluid intake control, regimented toilet regime, Oxybutinin, BoTox injection, intermittent self-catherterisation

Constipation:Difficulty opening bowels, or mobilising to toilet

  • Good diet and fluid, regular laxatives, bowel care, assistedevacuation, good hygiene

Other symptoms to be aware of:

  • Lhermitte’s symptom: ‘Electric shock’ felt down back of spine on neck flexion
  • Urthoff phenomenon:Symptoms worsening in the heat e.g. hot bath
127
Q

What are symptoms of transeverse myelitis?

A

Transverse myelitis =leg weakness, sensory changes, bladder/bowel disturbance

An area across (transverse) a section of spinal cord (myel-) that is inflammed (-itis)

  • May affect motor/sensory fibers or both. The types of fibres affected will produce the symptoms experienced by the patient.
  • May present as sensory changes (numbness, tingling) and/or motor changes (loss of power) below the level of the lesion. There is often a sensory level on examination. Signs are CNS not PNS.
  • Bladder, bowel and sexual function can also be involved.
  • The patient may feel a tight band-like sensation at the level of the lesion
128
Q

What is affected in brainstem inflammation as a result of mutiple sclerosis?

A

Brainstem = dizziness, vertigo, double vision

Inflammation to the brainstem or cerebellum. Many of the facial nerves have their origin in the brainstem. The head and neck sensation and power can be affected by inflammation in this area. This is rarer as a presentation of MS.

  • Vertigo and “room-spin” dizziness
  • Painless double vision as nerves controlling eye muscles are affected
  • Facial numbness
  • Dysarthria/Dysphagia

The brainstem co-ordinates the cranial nerves that control eye movement, to keep the eyes moving together smoothly. If it becomes inflamed, it can’t do it’s job properly and the eye movements become disjointed - leading to double vision for the patient as the brain tries to make sense of these disjointed images!

OnABductionof an eye (lateral rectus), we need to rapidly co-ordinateADDuctionof the other eye (medial rectus). To allow this to occur fast enough, a tract called themedial longitudinal fasciculuscrosses over to the contralateral side of the brain stem between the nucleus of the third and sixth cranial nerve. This tract is highlymyelinatedand often affected by MS.

When themedial longitudinal fasciculusis affected, there is a delay inADDuctionof the ipsilateral (same side as affected) eye when the contralateral eyeABducts. The term for this isinternuclear(between cranial nerve nuclei in the brainstem)ophthalmoplegia(eye movement paralysis).

129
Q

How can MS be investigated?

A

MultipleSclerosis must beMultipleEpisodes of demyelination and damage. These attacks must be disseminated in time and space. i.e. there must be attacks affectingdifferent areas, atdifferent times. Proof of this dissemination may be made clinically or via evidence on MRI scans.

Remember, an attack gradually worsens over a few days-weeks and must last at least 24hours.

The most common types of presentation are:

  • Optic neuritis
  • Transverse myelitis
  • BrainstemMRI

The 1st line investigation should be an MRI brain and spine to look for evidence of current and old inflammatory plaques.

The ‘classical’ plaques seen in MS are peri-ventricular. These appear on the T2 MRI images as white plaques very close to the ventricles. You can also see plaques in the cerebellum, brainstem or spinal cord. Plaques in different areas can show us evidence ofdissemination in space.

Adding gadolinium contrast to the MRI scan help us see active inflammation vs old inflammation. Active inflammation enhances with gadolinium contrast, old scarring doesn’t. This can show us evidence ofdissemination in time.

Oligoclonal bands:

These are evidence of inflammation and immunoglobulin synthesis. If they are foundonlyin the cerebrospinal fluid andnotin the blood then this suggests inflammationconfinedto the central nervous system not the rest of the body. CSF oligoclonal bands are found in MS, but also in other CNS inflammatory conditions such as encephalitis, sarcoidosis, lupus etc. This test issupportiveof an MS diagnosis ( if in keeping with a matching clinical history) but isnot specific.The presence of oligoclonal bands in suspected MS suggests longer-term inflammation so is sometimes used as evidence supportive ofdissemination in time.

Rule out other differential diagnosis:

If the presentation is atypical, or another cause is suspected, then tests may be performed to look for this. These may include a general bloods screen, specific autoantibodies, infection screens. These will likely be guided by an experienced neurologist based on the patient’s clinical presentation.

130
Q

How can MS be treated?

A

The aim of treatment for a MS relapse is to speed up recovery time. Treating an MS relapse does not impact on future disability or reduced risk of future relapses.

We treat relapses that cause symptoms that impact on a patient’s ability to function and perform their activities of daily living.

An MS relapse is treated with high dose steroids - to suppress the immune system and inflammation. This is why ruling out an infection first is so important, you don’t want to suppress the immune system of someone trying to fight off an infection!

MS relapses are treated with high dose IV or oral prednisolone for a few days.

DMTs aim to influence the progression of the disease. They do not cure the underlying disease itself. The aim is to:

  • Reduced the frequency of relapses
  • Reduce the progression of neurodisability

DMTs are givenearly,when the disease is in therelapsing-remittingphase.

The decision about when and what disease modifying treatment to start is not easy. There are different options available. This will usually be a shared decision between a specialist consultant and the patient.

You don’t need to memorise all of the medications available, the guidelines are changing frequentlyas the research progresses. It is enough to know that different options are available, and that these are a balancing act between efficacy and safety.

DMTs suppress the immune system. The stronger the DMT is, the more effective it is at reducing relapses. However, the risk of infections goes up. The condition that gets neurologists most worried is PML.

131
Q

What are the signs of Optic neuritis?

A

Optic neuritis is an inflammation of the optic nerve (cranial nerve III), reducing it’s ability to carry messages. This leads to a reduction in vision, particularly colour (especially red!) vision. Movement of the eye is often painful as the inflammed and swollen nerve moves. Jenny demonstrates an example of optic neuritis: painful, blurred vision unilaterally, which has gradually developed over several days.

Optic Neuritis is a common presentation of multiple sclerosis, but there are other causes.

Optic neuritis =blurred vision, painful eye movements

Inflammation and de-myelination of the optic nerve

  • Reduced visual acuity, particularly colour vision
  • Pain on eye movement
  • Relative afferent pupillary defect (RAPD) may be present
132
Q

How to assess an MS relapse?

A

The first step of assessment for MS relapse should always be torule out infection

Infection or fevers can lead to a temporary worsening of existing MS symptoms, as opposed to a new relapse with a new episode of inflammatory de-myelination.

The work-up for a possible MS relapse should always include looking for any signs of infection, particularly a UTI:

  • Thorough history and examination
  • Observations
  • Urine dipstick +/- culture
  • CXray if any respiratory symptoms/cough
  • Standard blood tests: FBC, U&Es, CRP

As well as infection, stress/heat/over exertion can also lead to a symptom deterioration. It is important to rule these factors out first, before jumping in to starting treatement.

An MRI can be useful to show evidence of a relapse if there is clinical uncertainty. Anewlesion, whichenhanceswith gadolinium contrast (showing active inflammation) is suggestive of an MS relapse. MRI is not always needed if clinical certainty is high.

133
Q

Name 5 blood test that a good initial screen for a patient presenting with fatigue, urinary and neurological symptoms

A

Full Blood Count

Urea and Electrolytes

Thyroid Function Tests

HbA1c

B12/folate

It is tempting to order a barrage of tests for ‘the fatigued’ patient, but often (especially in primary care) investigations need to be more targeted and specific. This would be a good starting point, but this list is not 100% definitive.

You might also think about a ferritin (more important in weight loss, change of bowel habit, any bleeding symptoms), Liver Function Tests (important in any GI, alcohol or bleeding issues), Bone panel (calcium, magnesium, phosphate) amongst others.

134
Q

What are signs of b12 deficiency?

A

B12 deficiency

A B12 deficiency can cause several different neurological symptoms. B12 is found in animal products and fortified cereals. People with poor intake of B12 (strict vegans without supplementation, poor nutritional diets) and those who cannot absorb B12 (lack of intrinsic factor due to pernicious anaemia, post-gastrectomy, Crohn’s disease at the terminal ileum etc.) are at risk of B12 deficiency. Body stores of B12 last for 2-4 years so it takes a long time to develop neurological symptoms from a B12 deficiency.

SACD

B12 deficiency can cause a peripheral neuropathy, affecting the distal limbs in a length-dependent pattern.

B12 deficiency can also cause a condition called ‘Sub-Acute Combined Degeneration of the Cord’ (SACD). This is a de-myelination of the fibres of the dorsal and lateral areas of the spinal cord.

Dorsal column: Vibration and Proprioception sense

Lateral corticospinal tract: A descending motor pathway

This means that SACD presents with a mix of Sensory and UMN motor signs. A patient may experience reduced sensation, reduced vibration/proprioception and ataxia as well as spastic loss of power and upgoing plantars (positive Babinski sign).

135
Q

What can cause a raised MCV?

A

B12 deficiency,.

Folate deficiency

Hypothyroidism:

Alcohol: Alcohol is another common cause of a raised MCV,

Acute illness/bone marrow stress: sepsis/trauma/bone marrow dysplasia conditions

136
Q

What can trigger GBS?

A

Supporting this theory, 2/3 of GBS patients have an upper respiratory tract infection or gastroenteritis (viral or bacterial) in the 6 weeks before disease onset2. The most commonly identified trigger is aCamplyobacter jejuniinfection, hence this patient’s preceding diarrhoea. Furthermore, antibodies to specific gangliosides (compounds found mostly in the nervous system) are strongly associated with different variants of GBS.

137
Q

How can GBS present?

A

It is also worth briefly noting at this point that there are multiple other variants of GBS, which you do not need to know in detail. These include variants that cause only sensory symptoms, axonal injury instead of demyelination, and a variant called Miller-Fischer syndrome that causes eye weakness (ophthalmoplegia).

Classically, patients with AIDP present with mild paraesthesia of the lower limbs. This is followed by progressive weakness that starts in the lower limbs and progresses to the upper limbs. Patients develop a flaccid paresis with areflexia. This is due to demyelination of lower motor neurons. Despite the paraesthesia, patients often have no sensory deficit on examination. There is usually back or leg pain due to nerve root inflammation.

patient has wasting as a long-term consequence of GBS. This occurs in a minority of patients. You would not see wasting in your patient acutely.

Autonomic symptoms can occur due to inflammation of autonomic ganglia. However, bladder/bowel symptoms tend to occur later on, if at all. Therefore, early symptoms would make you more suspicious of spinal cord lesions / cauda equina. Other autonomic symptoms include sudden changes in BP and HR, as well as arrhythmias (rarely).

The course of the disorder is variable but the majority of patients have reached the worst point of their illness by 2 weeks. This is followed by a plateau phase, which is very variable. The majority of patients make a dramatic recovery after their plateau and at least 28 days after onset. Most patients have a return to daily function but are left with some form of residual deficit e.g. fatigue, weakness and pain. 20% of patients are still unable to walk at 6 months and there is a mortality of 3-10%.3

Mortality is largely due to the significant risk of respiratory muscle involvement (40%) and neuromuscular respiratory failure (25%)4. Therefore patients need to have regular forced vital capacity (FVC) assessments. This is the pulmonary function test in which the patient exhales their maximum volume of air, following a maximal inspiration. Poor / worsening FVC has been found to be the most useful prognostic indicator of the need for ventilation (covered in the treatment section). Observations that might usually be helpful for indicating respiratory failure, such as dropping O2 saturation, occur much later in GBS.

138
Q

What investigation can be done to diagnose GBS ?

A

Routine blood tests for patients with GBS are often normal but they may have mildy raised CK and abnormal LFTs. It is useful to check for inflammatory markers, which should not be raised (GBS occurs after the infection), and biochemical markers that can affect nerve function (e.g. hypocalcaemia). Anti-ganglioside antibodies can be tested for specific GBS variants if the diagnosis is unclear.

Lumbar puncture in these patients is useful and should show ‘albuminocytologic dissociation’. This means that there is elevated protein in the cerebrospinal fluid (CSF) but little or no cells (cytologic). In other words, there aren’t any cells in the CSF to account for the high protein. It is not fully understood why this is the case. You do not need to memorise these numbers but the cell count is usually less than 50 and the protein is greater than 0.55 g/L.

The other useful tests are nerve conduction studies (NCS) / electromyography (EMG). These tests measure the response of nerves and muscles respectively to electrical stimulus. This essentially allows us to localise the pathology to the peripheral nerves and note acquired patchy demyelination.

It should be noted that these abnormal signs on lumbar puncture and NCS/EMG are present in more patients 7 days after presentation than when tested earlier. It is therefore possible to do these investigations too early and be falsely reassured by results. You would discuss these decisions with a neurologist.

139
Q

How can GBS be treated?

A

The main treatment for GBS is either plasma exchange (PLEX) or intravenous immunoglobulin. Both of these treatments have similar efficacy in GBS and are not beneficial in combination.

Plasma exchange (AKA plasmapheresis), is a therapy by which blood is extracted from the patient from a large vein (via a line) and the plasma is separated from other blood contents. The patient’s autoantibodies are contained within the plasma as so this part of of the blood is discarded. The patient’s remaining blood products (RBCs, WBCs and platelets) are mixed with donor plasma or an albumin/saline solution and returned to the patient.

Intravenous immunoglobulin (IVIg) is a therapy by which high dose donor immunoglobulin is given intravenously to the affected patient. Exactly why this works is not known but IVIg appears to neutralise the effect the patient’s autoantibodies and modulate the immune response.7

Both of these treatments are effective in improving recovery time, admission time, time to walk again unaided and reducing the likelihood of the patient requiring ventilation. Steroids have not been found to be useful for treating GBS.

is also important to consider supportive management of the patient. We have mentioned that there can be autonomic dysfunction in these patients. Changes to heart rate and blood pressure can be treated, if necessary, with short acting medication as we know that these changes are labile. Patients may need catheterisation if they develop urinary retention. They are highly likely to develop constipation due to immobilisaton, even without autonomic dysfunction. Therefore laxatives are often prescribed. Immobilisation also increases the risk of venous thromboembolism and these patients usually require prophylactic dose anticoagulation until their mobility improves. Some patients also need nutritional support by tube feeding.

Patients with an FVC of less than 20ml/Kg are associated with progression to respiratory failure and intensive care admission should be considered. This is for invasive / mechanical ventilation to provide the alveoli with new air to undergo gaseous exchange. Giving high concentration oxygen alone is less helpful here because this in not reaching the alveoli.

140
Q

What are the different Parkinson syndromes and how do they present?

A

Parkinsonism as a description of a collection of signs:

  • Bradykinesia (slowness of movement and decrement on rapid, alternating movements)

+ at least one of:

  • Rigidity (muscle stiffness)
  • Postural instability
  • Tremor

Idiopathic Parkinson’s Disease is the most common cause of Parkinsonism, but there are other causes. These are explored further in the lecture on Parkinson’s disease. Drug-induced Parkinsonism (think anti-pyschotics), Vascular and the ‘Parkinsonian plus’ conditions are possibilities.

There are a fewkey featuresin the history that could help us differentiate between the Parkinsonian Plus conditions.

Progressive Supranuclear Palsy:

  • Earlybackwardsfalls
  • Gaze palsy, particularly on looking vertically
  • Axial (body) rigidity
  • May have more frontal lobe signs

Multiple Systems Atrophy:

  • Earlysevere autonomicdysfunction
  • Postural hypotension, may lead to faints
  • Urinary symptoms, may lead to incontinence
  • Signs may be more Parkinsonian, or more cerebellar

Corticobasal Degeneration:

  • Very rare
  • Asymmetrical signs,one limb most affected
  • Cortical sensory loss - ‘alien limb syndrome’, dyspraxia, shape recognition

Lewy Body Dementia:

  • Dementiaprecedesthe Parkinsonian features
  • Fluctuates

Visual hallucinations

  • often children/animals

In the past we used to think of PD as a condition of “shaking and slowness”. James Parkinson’s original descriptions were entitled “the shaking palsy”. As we have better understood the condition we have to learned to see it as a much more ‘multi-system’ illness. We tend to split the symptoms into motor (relates to muscles and movement) and non-motor (everything else).

We are starting to recognise that there are 2 main sub-types of Parkinson’s Disease:

  • Tremor dominant
  • Postural Instability and Gait Disturbance (PIGD)

Non-motor symptoms may become more prominent over time as the disease gradually progresses.

141
Q

What are the non-motor symptoms of Parkinson’s?

A

The non-motor symptoms are numerous and varied:

Balance and falls:

  • Postural instability (poor balance)
  • Postural hypotension
  • Eyesight problems (dry eyes, blurred or double vision)

Bladder and bowel

  • Constipation
  • Gastroparesis
  • Urinary symptoms (urgency, frequency, nocturia)
  • Erectile dysfunction

Speech and swallow

  • Dysphagia
  • Excess salivation/drooling
  • Hypophonia (quiet voice)
  • Anosmia (loss of sense of smell, often pre-dates the physical symptoms)

Sleep

  • Insomnia (fragmented sleep, rather than trouble falling asleep)
  • REM sleep disorder (Loss of usual paralysis during sleep, leading to vivid dreams and acting out their dreams e.g. punching a bed partner!)
  • Restless legs syndrome

Mood and cognition

  • Cognitive impairment
  • Anxiety
  • Depression
  • Apathy (remember, this is different to depression. Initiation/planning is reduced, but the patient can still enjoy things and do not necessarily have low mood)
  • Visual hallucinations
  • Psychosis
  • Pain
142
Q

How to give Parkinson’s meds if a patient is nil by mouth?

A

. If you are unsure, discussing with your team is always your best option. Always try to give medications in their oral form if possible, ‘Nil By Mouth’ sometimes means small sips or tablets can be taken. Always check with the nurses and SALT team about the ‘NBM’ order. Do not give anything orally unless you are sure and have discussed this. If this patient is going to be NBM for a prolonged period they will need some form of nutrition so may need an NG tube for feeding. Medications can also be given via an NG tube.

  1. If the oral route is unavailable then medication patches are the next best option. Rotigotine is a dopamine agonist that can be given in 24 hour patch form. Check the dose conversion using online resources (e.g. OPTIMAL calculator) and ask the ward pharmacist to assist and check your conversions. Remember, the conversion is not perfect and some patients may experience side effects with a dopamine patch. Always give their usual medication in oral form if possible.
  2. Withholding medication is not a good option. Missing doses of medication leads to deterioration of symptoms and patient compromise. The swallow is likely to be worse after several days without medication. Sudden withdrawal of medication can also lead to Neuroleptic Malignant Syndrome.
  3. Levodopa does not come in an IV form, this is an actively dangerous and unsafe option so is ranked as the worst choice.

If you are unsure about PD medications the best option is to refer the patient and ask for help! Informing the SALT team and pharmacist is a good place to start. Remember, the best option is always their usual medication, in the usual form at the usual times. If that’s unavailable, then consider nasogastric tube placement or a transdermal patch. Never omit Parkinson’s medications!

143
Q

How is Parkinson’s treated and what are the side effects of medication?

A

In younger patients with few medical co-morbidities it is often a good idea to start with a dopamine agonist or MAO-B inhibitor. This allows us to delay starting levodopa and avoid early problems with prolonged levodopa use.

Older patients with more co-morbidities +/- frailty will often be less able to tolerate the side effects of dopamine agonists. They will tend to be started on Levodopa as a first line agent and low doses of other medications can be added later if tolerated to keep symptoms under control.

Levodopa Co-beneldopa (Madopar), Co-carledopa (Sinemet)
Dopamine agonists Pramipexole, Ropinirole, Rotigotine, Apomorphine
MAO-B inhibitors Rasagiline, Selegiline
COMT inhibitors Entacapone
Amantidine- Dyskinesia

Levodopa often works very effectively for the first 5-7 years of use, sometimes referred to as the ‘Honeymoon’ period’. After this time some patients will start to experience motor fluctuations where the dose of levodopa doesn’t offer consistent control of their symptoms throughout the day.

This causes three main issues:

  • Wriggling:Peak-dosedyskinesia (increased movement e.g. twitching, writhing, jerking, twisting) as levodopa reaches it’s maximum concentration in the blood 1-2 hours after the dose is taken
  • Wearing off:“Wearing off” effect whereParkinsonian symptoms (rigidity, bradykinesia, tremor etc) start to re-occur before the next dose is due, when levodopa is at it’s minimum concentration in the blood
  • Waiting:“Delayed on” effect where it takes an increased time for levodopa to start having an effect after the tablet is taken.

Over time the number of doses may have to be gradually increased to keep giving the same benefit to the patient. The risk of developing these medication issues increases with prolonged and high dose therapy.

Dopamine Agonists

Dopamine agonists are useful medication as an early alternative to Levodopa or in combination, to allow lower dosages to be used. As with all medications, consideration must be given to their unwanted side effects.

  • Postural hypotension
  • Hallucinations
  • Poor sleep and daytime solomnence
  • Impulse control disorders: Patient’s can start to experience issues with gambling, excess shopping, hypersexuality or binge eating

It is important to warn patients to watch out for these side effects when starting any new medications or changing doses!

144
Q

Who is eligible for DBS?

A

Some patients think that Deep Brain Stimulation (DBS) offers a ‘miracle cure’ for their condition and will ask their doctor about having one inserted. Whilst DBS can offer huge benefits to symptom control for some patients, sadly, it is not appropriate for everyone.

The main points to be aware of are:

  • Works mostly on motor symptoms (eg. tremor) rather than non-motor.
  • Symptoms need to be dopamine responsive (i.e. a proven benefit on dopaminergic medications) to know that DBS could help.
  • DBS can make some things worse: Balance, eye lid opening, speech, swallow and cognition can be worsened by DBS. Anyone with pre-existing issues in these areas would not be appropriate for DBS.

Going for DBS is a significant undertaking for patients and it must be a shared decision between patient and clinician after consideration of the risks and benefits. Have a look at this video to see what can be possible when things go well

145
Q

What is Lateral medullary syndrome?How can it be differentiated from Weber’s syndrome

A

Lateral medullary syndrome, also known as Wallenberg’s syndrome, occurs following occlusion of the posterior inferior cerebellar artery.

Cerebellar features
ataxia
nystagmus

Brainstem features

ipsilateral: dysphagia, facial numbness, cranial nerve palsy e.g. Horner’s
contralateral: limb sensory loss

Weber’s syndrome (branches of the posterior cerebral artery that supply the midbrain)
Ipsilateral CN III palsy
Contralateral weakness of upper and lower extremity

Other Posterior Stroke Syndromes

Basilar artery occlusion is more likely to present with locked in syndrome (quadriparesis with preserved consciousness and ocular movements), loss of consciousness, or sudden death.

Anterior inferior cerebellar artery results in lateral pontine syndrome, a condition similar to the lateral medullary syndrome but with additional involvement of pontine cranial nerve nuclei.

146
Q

What are 4 clinical groups of motor neurone disease?

A

Motor neuron disease can be classified clinically and pathological according to the distribution of motor neuron involvement.

As present our characterisation is limited to distinguishing sporadic from familial disease, and division into four clinical groups.

The most common variant, known as amyotrophic lateral sclerosis, affects both upper and lower motor neurons and therefore manifests as a mixed clinical picture of upper and lower motor neuron signs.

The four clinical groups are:

Spinal ALS (the classic MND syndrome)

Bulbar ALS (with early tongue and bulbar involvement)

Progressive muscular atrophy (with only lower motor neuron features)

Primary lateral sclerosis (with only upper motor neuron features).

147
Q

What are the clinical features MND and how is it diagnosed and treated?

A

Clinical features

Early bulbar and respiratory muscle involvement, and older age at onset carries the poorest prognosis. In contrast, more prominent lower motor neuron features is associated with more protracted survival.

Classically, there is a combination of upper motor neuron and lower motor neuron signs.

Upper motor neuron signs include spasticity, hyperreflexia and upgoing plantars (though they are often down going in MND).

Lower motor neuron signs include fasciculations, and later atrophy.

Generally, the eye and sphincter muscles are spared until late in the disease course and sensory disturbance is NOT seen (and should prompt consideration of an alternative diagnosis).

The diagnosis of motor neuron disease is clinical, but nerve conduction studies will show normal motor conduction and can help exclude a neuropathy. Electromyography shows a reduced number of action potentials with increased amplitude. MRI is usually performed to exclude the differential diagnosis of cervical cord compression and myelopathy

*vague sensory symptoms may occur early in the disease (e.g. limb pain) but ‘never’ sensory signs

Treatment of motor neuron disease

Currently there is only one disease modifying treatment available for motor neuron disease, and its results are modest at best: Riluzole (an antiglutamatergic drug which dampens motor nerve firing) has been shown to prolong life by 3 months.

Non-invasive ventilation can also prolong survival in patients with type 2 respiratory failure.

Treatment focus is therefore supportive and best coordinated via and MDT approach.

Pain relief is important, with simple analgesia as well as the treatment of spasticity and contractures with baclofen and botox injections.

Drooling may be helped by anticholinergics (including the TCAs), and eventually supportive feeding via an NG or PEG may be indicated as bulbar disease progresses.

Early discussion of advanced care planning should also be initiated to minimise distress and complications as disease progresses.

148
Q

What are the features and important clinical associations of Myasthenia graves?

A

Myasthenia gravis is an autoimmune disorder resulting in insufficient functioning acetylcholine receptors. Antibodies to acetylcholine receptors are seen in 85-90% of cases*. Myasthenia is more common in women (2:1)

The key feature is muscle fatigability - muscles become progressively weaker during periods of activity and slowly improve after periods of rest:
extraocular muscle weakness: diplopia
proximal muscle weakness: face, neck, limb girdle
ptosis
dysphagia

Associations
thymomas in 15%
autoimmune disorders: pernicious anaemia, autoimmune thyroid disorders, rheumatoid, SLE
thymic hyperplasia in 50-70%

149
Q

What Investigations are done for myasthenia graves and how is it managed?

A

Investigations
single fibre electromyography: high sensitivity (92-100%)
CT thorax to exclude thymoma
CK normal
autoantibodies: around 85-90% of patients have antibodies to acetylcholine receptors. In the remaining patients, about about 40% are positive for anti-muscle-specific tyrosine kinase antibodies
Tensilon test: IV edrophonium reduces muscle weakness temporarily - not commonly used any more due to the risk of cardiac arrhythmia

Management
long-acting acetylcholinesterase inhibitors
pyridostigmine is first-line
immunosuppression:
prednisolone initially
azathioprine, cyclosporine, mycophenolate mofetil may also be used
thymectomy

Management of myasthenic crisis
plasmapheresis
intravenous immunoglobulins

150
Q

What is Lambert Eatons disease and how is it diagnosed and managed?

A

Lambert-Eaton myasthenic syndrome is seen in association with small cell lung cancer and to a lesser extent breast and ovarian cancer. It may also occur independently as an autoimmune disorder. Lambert-Eaton myasthenic syndrome is caused by an antibody directed against presynaptic voltage-gated calcium channel in the peripheral nervous system.

Features
repeated muscle contractions lead to increased muscle strength (in contrast to myasthenia gravis)
in reality, this is seen in only 50% of patients and following prolonged muscle use muscle strength will eventually decrease
limb-girdle weakness (affects lower limbs first)
hyporeflexia
autonomic symptoms: dry mouth, impotence, difficulty micturating
ophthalmoplegia and ptosis not commonly a feature (unlike in myasthenia gravis)

EMG
incremental response to repetitive electrical stimulation

Management
treatment of underlying cancer
immunosuppression, for example with prednisolone and/or azathioprine
3,4-diaminopyridine is currently being trialled
works by blocking potassium channel efflux in the nerve terminal so that the action potential duration is increased. Calcium channels can then be open for a longer time and allow greater acetylcholine release to the stimulate muscle at the end plate
intravenous immunoglobulin therapy and plasma exchange may be beneficial

151
Q

What are the different types of Interstitial lung disease?

A

exposure related-drug Induced(chemotherapy,Methotrexate,Amiodarone)
Occupational(fumes, organic,Inorganic fumes)
Hypersensitivity

Autoimmune relayed-RA,SLE,Scleroderma,Polymyositis

Idiopathic-IPF,Acute interstitial pneumonia

Examples
Pulmonary fibrosis,Hypersenstivity Pneumonitis,Sarcoidosis,Pneumoconiosis,

152
Q

What important to elict in the history and examination on Pulmonary fibrosis?

A

History-Autoimmune disease, job,occupation,pets,drugs(methotrexate, Nitrofurantoin,Amiradone)

Common features of IDL include:

  • Cough - dry and persistent
  • Breathlessness - secondary to poor lung compliance
  • Cyanosis
  • Crackles - typically bilateral and basal, typically fine crackles often described as “velcro-like”
  • Clubbing - common in idiopathic pulmonary fibrosis, pathophysiology not fully understood, also seen in other conditions
  • Exertional hypoxemia - due to maldistribution of ventilation and perfusion

An examination of the cardiac system is also helpful, looking for signs of left and right heart failure.

Other non thoracic signs may also be present if the ILD is a result of a systemic illness eg. rash in sarcoidosis.

153
Q

What are the signs of IPF?

A

Idiopathic Pulmonary Fibrosis (IPF) is the most common of the idiopathic pneumonias. There is no evidence of an alternative cause eg. drugs, environmental exposure or a system condition.

It is chronic and progressive, and can be more common in males.

Clinical Features:

  • Insidious onset of shortness of breath and cough
  • Exertional dyspnoea
  • Malaise
  • Weight loss
  • Arthralgia
  • Fever

Often patients will be diagnosed as recurrent chest infections before the diagnosis of IFP is considered.

Important things to look for in the examination are

Fine end-inspiratory crackles are oft described as velcro crackles

A thorough examination of the cardiovascular system is also recommended

154
Q

What are the investigation findings of IPF?Including lung findings?and treatment

A

Investigations for IPF should include

  • ABG
  • Autoantibodies and serum ACE (to rule out other known causes)
  • CXR
  • Lung Function Tests
  • CT/High Resolution CT

Spirometry showing Obstructive and Restrictive Lung Disease([Source:By CNX OpenStax [CC BY 4.0)

Lung function tests in IPF with show a restrictive pattern:

  • Reduced FEV1 (<80% of the predicted normal)
  • Reduced FVC (<80% of the predicted normal)
  • FEV1/FVC ratio normal (>0.7)
    In pulmonary fibrosis-total lung capacity reduced

This ABG shows Type one respiratory failure which is typical of IPF.
CXR findings: Decreased lung volume, bilateral lower zone reticulonodularshadows and honeycomb lung in advanced disease

CT findings: Diffuse reticular infiltration of both lungs associated with honeycombing

Treatment
Approach to management in IPF:

Conservative

  • Best supportive care
  • Oxygen
  • Pulmonary Rehab
  • Palliative care when appropriate

Medical

  • Perfenidone - anti-fibrotic properties, can stabilize lung function
  • Nintedanib

Surgical

  • Lung transplant
155
Q

What are the Indications for Cardiac CT?

A
Post CABG assessment
Assessment of Aorta
Assessment of pulmonary vessels
Congenital anomalies of the heart
Assessment of  prosthetic valves
Assessment of cardiac masses
Assessment of pericardium
156
Q

What does Cardiac CT include?How is Calcium scoring done?

A

Includes
Cardiac Calcium scoring (without contrast)
Cardiac Angiography (with Contrast)
To begin with a cardiac calcium scoring is done without contrast. Depending on the score, further need for angiography is decided.
E.g.. If the score is 0 or >400: No need for further Angiography.
If intermediate, then proceed with angiography.

Calcium scoring
It is done without a contrast.
It measures whether calcified atherosclerotic plaque is present in the vessels or not.
Based on the principle that
Obstructive atherosclerotic plaques are calcified.
Calcium is not present in the walls of a normal coronary artery.
It is a semi-automated measure. We have to look at the coronary arteries, one by one and click on the visible hyperdensity(calcium), and a automated score is obtained . The scores of all the vessels are added up to get a final score.

157
Q

What are the Advantages and disavantages of Cardiac CT?

A

Advantages

Non invasive
Fast
Calcium scoring
3D reconstruction
Can see beyond the lumen(atheroma imaging)

Disadvantages

Contrast requirement(adequate renal function)
Limited spatial resolution
Radiation
Requires slow heart rate(preferably<60BPM)
No hemodynamic information
Movement artefacts
Limitation if high amount of calcium is present in the vessels.
Obese and Unco-operative patient
Stents

158
Q

What are the Indication and Contraindications for cardiac transplant?

A

Indication

Cardiogenic shock
HF stage 3 /4
progressive symptoms with maximal therapy
Severe symptomatic hypertrophic and restrictive cardiomyopathy
Cardiac masses non metasisi
hypo plastic left heart syndrome and congenital heart diseases
Life threatening arrhythmia
Medically refractory angina

Contraindications

Severe Pulmonary HTN
>6 Wood units not responsive to vasodilators
Active infection
Uncontrolled Malignancy
Irreversible end-organ disease
Hepatic, Renal, Pulmonary
Pulmonary Infarction
Age >60 years
Diabetes mellitus with end-organ damage
Severe Cerebral and Peripheral Vascular Disease
159
Q

What are the types of Cardiac transplant?

A

All recipients are given Cyclosporine f or immunosuppression
Induction may be difficult due to failing heart
Must keep heart rate high to maintain higher C.O. with patients with low stroke volume
Surgical Techniques;

Orthotopic Heart Transplant- (Most Common)
Native heart completely removed - and replaced
Contraindicted in PTN
Biatrial anastomosis-shorter schema time, size mismatch,arrythmias

Bicaval anastomosis-narrowing SCV,IVC ,less arrhythmia

Heterotopic Heart Transplant-
Donor heart placed in parallel with native – two heartsin
REVERSAL hf
Pros-
Native heart gives the donor heart a helpful kick
Offers some protection against rejection of the donor heart
Cons-
Decreased survival rate
Still have to medically manage a failing native heart
Native heart is a site of thrombus

160
Q

Whats the long term management following heart transplant?

A

check notion

161
Q

What post operative complication can occur following a cardiac transplant?

A

Check notion

162
Q

What are the causes of Primary and secondary Cardiomyopathy?

A

Primary

Pathology predominantly involves the heart
Examples:
Dilated cardiomyopathy
Hypertrophic cardiomyopathy
Restrictive cardiomyopathy
Arrhythmogenic right ventricular cardiomyopathy
Obliterative cardiomyopathy

Secondary
Autoimmune-Dermatomyositis
Rheumatoid arthritis
Scleroderma
Infection-Chagas disease
HIV
Hepatitis C
Rickettsia
Viral (adenovirus, Coxsackie, Epstein-Barr, parvovirus)
Endocrine-Acromegaly
Diabetes mellitus
Hyperthyroidism
Hypothyroidism
Hyperparathyroidism
Obesity
Toxins-Alcohol
Anabolic steroids
Chloroquine
Heavy metals (arsenic, cobalt, lead)
Haemochromatosis
Stimulants (cocaine)

others
Amyloidosis
Gaucher disease
Hunter Syndrome

Kwashiorkor
Niacin deficiency

Glycogen storage disorders
Neurofibromatosis
Muscular dystrophy (Becker’s, Duchenne, myotonic)

163
Q

How does Dilated Cardiomyopathy present?how is it diagnosed and managed?

A

Autosomal dominant

Other causes-Cox sackie B,DoxirubinAlchol,Beri beri

Clinical-Fatique,dyspnea,PE,RVF,AF

Sign-increase pulse, bp,JVP,DISPLACED APEX,s3 gallop, mitral tricuspid regurgitation,jaundice,hepatomegaly

Prevalence 1:2500
May present with:
Heart failure
Cardiac arrythmia
Conduction defects 
Thromboembolism
Sudden death
Family history should be obtained 
inx
BNP
ECG
low sodium-poor prognosis
Cardiac MRI
Coronary angiography
To exclude coronary artery disease 

Endomuocaridal biopsy
Acute dilated cardiomyopathy + refractory heart failure symptoms
Dilated cardiomyopathy in presence of systemic diseases
SLE, Polymyositis, Sarcoidosis
Rapidly progressive ventricular dysfunction
In unexplained, recent onset cardiomyopathy
New onset cardiomyopathy + recurrent VT/ high grade heart block
Heart failure in the setting of fever, rash and peripheral eosinophilia

Management
Manage symptoms of cardiac failure-ACE1,diuretics,beta blockers
Implantable Cardioverter Defibrillator (ICD),LVAD
Cardiac transplant

164
Q

What is Hypertrophic Cardiomypathy?What can cause this?How can it be classified?

A

Left ventricular hypertrophy
In absence of causative haemodynamic factors:
Hypertension, aortic valve diseases, systemic diseases, storage diseases
Prevalence 1:500-1:1000
Men : women = 2:1
African-Americans : Caucasians = 2:1
Commonly reported in North America, West Europe, Asia

Autosomal dominant with incomplete penetrance
Mutation in genes encoding for sarcomere proteins
Myosin, actin, troponin, tropomyosin
Genetic basis does not directly correlate with prognostic risk stratification

Classified into obstructive and non obstructive

165
Q

What is the pathophysiology of HOCM?

A

Left Ventricular Outflow Obstruction

(Asymmetrical hypertrophy of interventricular septum (ASH)
Systolic anterior motion (SAM) of mitral valve
Leaflets of mitral valve move towards enlarged septum during systole
Mitral valve leaflets obstructs blood flow from ventricles to aorta
Narrowing of outflow tract
Major determinant for heart failure symptoms, and death )
Elevated intraventricular pressure
Prolonged ventricular relaxation
Leads to diastolic dysfunction
Increased myocardial wall stress
Increased bodily oxygen demand
Decreased cardiac output

Mitral regurgitation

(Result from systolic anterior motion of mitral valve
Severity directly proportional to outflow obstruction
Patients complain of dyspnoea and orthopnoea )

Diastolic dysfunction

(Impaired ventricular relaxation
High systolic contraction load
Contraction/ relaxation of ventricles not uniform
Exertional dyspnoea symptoms
Abnormal diastolic filling
Increased pulmonary venous pressure )

Myocardial ischemia
(Often occurs without atherosclerotic coronary artery disease
Postulated mechanism:
Abnormally small +/- partially obliterated intramural coronary arteries
Due to hypertrophy
Inadequate number of capillaries for the degree of mass )

166
Q

How can HOCM present, be investigated and managed?

A
Symptoms*
Chest pain
Dyspnoea
Syncope with exertion
Pulmonary oedema
Cardiac arrhythmias
e.g. atrial fibrillation 
Sudden death

Signs- Palpation of pulses
Double apical pulsation
Forceful atrial contraction
Jerky carotid pulse
Short upstroke, prolonged systolic ejection
Jugular venous pulse
Prominent a wave: decreased ventricular compliance

Signs- Auscultation
Fourth heart sound (S4)
Cardiac hypertrophy
Ejection systolic murmur
Along left lower sternal border
Intensity increase with decreased preload (Valsalva manoeuvre), vice versa (during squatting) 
Mitral regurgitation
Secondary to SAM (systolic anterior motion)
INX-
ECG(Left ventricular hypertrophy
Presence of septal Q waves)
Echocardiography
(Used for:
Diagnosis
Haemodynamic assessment
Clinical risk stratification
Interventional management
Findings:
Left ventricular hypertrophy (LVH)
Asymmetrical septal hypertrophy (ASH)
Systolic Anterior Motion (SAM)
)LVH ≥15mm (Asymmetric > Symmetric)
Absence of other cardiovascular/ systemic disease associated with LVH/myocardial wall thickening 

Cardiac Magnetic Resonance (CMR)

Indicated when ECHO views are limited
Due to unusual distribution of hypertrophy
Detect milder magnitudes of hypertrophy
Demonstrates myocardial scarring
Differentiate hypertrophic cardiomyopathy from other LVH
Gadolinium enhanced imaging detect myocardial scarring in ~2/3 of patients with hypertrophic obstructive cardiomyopathy

Cardiac Catheterisation
Not typically necessary in HCM
Hyperdynamic systole function results in almost complete obliteration of left ventricular cavity

Management

Aims:
Treat symptoms
Prevent sudden death

Options:
Medications
Dual-chamber pacing
Implantable Cardioverter Defibrillator (ICD)
Surgical septal myomectomy
Alcohol septal ablation 

Beta-blockers
During chest pain
Increase ventricular diastolic filling/ relaxation
Decrease myocardial oxygen consumption
Not been shown to reduce incidence of sudden cardiac death
Verapamil
During chest pain
Augments ventricular diastolic filling/ relaxation

167
Q

What are the complications and indication for ICD in hypertrophic cardiomyopathy?

A

notion

168
Q

What is Arrhythmogenic right ventricular Cardiomyopathy and how does it present?

A
Predominantly affects right ventricle
Population prevalence 1 : 5000
Autosomal dominant inheritance 
Fatty/ fibro-fatty replacement of myocytes
Leads to ventricular dilation
Clinical
Most patients asymptomatic
Symptoms (if present):
Ventricular arrhythmia
Syncope
Right heart failure 
Increased risk of sudden death
169
Q

How can ARVC be Investigated and managed?

A

inx

ECG
Usually normal
May show T-wave inversion in right ventricular leads (V1, V2)/ features of RBBB
24-hours Holter monitoring
Non-sustained ventricular tachycardia
Echocardiography
Right ventricular dilation + aneurysm formation (in advanced cases)
Cardiac Magnetic Resonance
Fibrofatty infiltration 
Treatment
Amiodarone
Symptomatic arrhythmias
Beta blockers
Non-life threatening arrhythmias
Implantable Cardioverter Defibrillator (ICD)
Life threatening arrythmias
Cardiac transplant 
Intractable arrhythmia
Cardiac failure
170
Q

What is Left Ventricular non compaction?

A

Prominent trabeculations, deep recesses in apex of left ventricle
Thin compact epicardium, thickened endocardium
Increased risk of thrombosis, heart failure, ventricular tachycardia and sudden death
Risk of offspring inheriting disease

171
Q

How does Alcholic Cardiomyopathy occur?

A

1/3 of dilated cardiomyopathy cases in the Western world
Risk increases with alcohol consumption >10 years
Advice early alcohol abstinence
Chronic, insidious onset
Result of
Direct toxin from alcohol
Thiamine deficiency

Clinical
Presentation:
Dyspnoea, orthopnoea, paroxysmal nocturnal dyspnoea
Palpitation
Syncope
Chest pain
Pleural effusion
ECG findings:
Atrial fibrillation/ Atrial flutter
QT prolongation
Poor R-wave progression
Premature ventricular contraction
LBBB/RBBB
1st degree atrioventricular block
172
Q

What is peripartum Cardiomyopathy?

A

Systolic heart failure (LVEF <45% / fractional shortening <30%)
In the last month of pregnancy/ within 5 months of delivery
Other determinable aetiology of heart failure absent
Absence of any heart disease before last month of pregnancy

Fractional shortening:
Reduction of length of end-diastolic diameter by the end of systole
Measures heart’s muscular contractility
Efficiency of heart in ejecting blood impaired when diameter fails to shorten by >28%

173
Q

How can inherited infiltrative diseases causes Cardiomyopathy?

A

Haemochromatosis:
Ventricular and cardiac conduction system involvement
Complications: ventricular wall thickening, dilated cardiomyopathy, restrictive cardiomyopathy
Fabry disease:
Glycolipid deposition in endothelium, myocardium, mitral valve
Complications: hypertension, mitral regurgitation, heart failure
Gaucher disease:
Cerebrosides (glycosphingolipids) deposition
Complications: left ventricular dysfunction, haemorrhagic pericardial effusion

174
Q

What is Restricitve Cardiomypathy and what are the causes?

A

Diastolic insufficiency
Impaired ventricular filling
Ventricles stiff and rigid
Increased tension of ventricular filling
Systolic function normal in early stages
Intraventricular pressure rises precipitously with small increase in volume

Causes

Primary restrictive cardiomyopathy
Idiopathic
Loeffler eosinophilic endomyocardial disease (for your interest) 
Secondary restrictive cardiomyopathy
Amyloidosis
Sarcoidosis
Haemochromatosis

Morphology

Hypertrophy of myocardium
Myocardium becomes firm 
Thickening of endocardium
Both atrium dilated 
Due to diastolic disturbances 
Ventricle size slightly enlarged
No dilation of cavities 
Ventricle walls loss elasticity

Under microscope -Patchy/ diffuse interstitial fibrosis/ amyloid

175
Q

How to differentiate restrictive cardiomyopathy from constrictive pericarditis?

A

notion

176
Q

How can Restrictive Cardiomyopathy present?how can it be Investigated and managed?

A

clinical

Symptoms
Fatigue
Dyspnoea
Tachypnoea

Examination findings
Increased JVP
Hepatic enlargement
Ascites
Oedema 
S4 heart sounds 

Inx

Examination findings

Increased JVP
Hepatic enlargement
Ascites
Oedema 
S4 heart sounds 
Echocardiography
Symmetrical myocardial thickening
Impaired ventricular filling 
Cardiac Magnetic Resonance
Myocardial fibrosis in amyloidosis
Endomyocardial biopsy

Management
No specific treatment
Manage cardiac failure and embolic manifestations
Cardiac transplant in selected cases
Melphalan, prednisolone, colchicine may improve survival in primary amyloidosis

177
Q

What is Obliterative Cardiomypathy?

A
Rare form of restrictive cardiomyopathy 
Thrombosis + fibrosis + obliteration of ventricular cavities
Involves endocardium of one or both ventricles
Endocardium thickens 
Features:
Mitral + tricuspid valve regurgitation
Heart failure
Pulmonary embolism
Systemic embolism 

Associated with:
Eosinophilic leukaemia
Chrug-Strauss syndrome

Management:
Anticoagulants + antiplatelets
Manage heart failure
Tricuspid and mitral valve replacement +/- decortication of endocardium